Как делать раствор: Цементный раствор: пропорции, приготовление, состав, характеристики

Как делать раствор: Цементный раствор: пропорции, приготовление, состав, характеристики

Содержание

Цементный раствор: пропорции, приготовление, состав, характеристики

Цемент – один из основных строительных материалов. Представляет собой сухое порошкообразное вещество неорганического происхождения. Этот вяжущий материал при взаимодействии с водой образует пластичную смесь, которая при затвердевании превращается в камневидное тело.

Наиболее распространенная разновидность цемента – портландцемент, в состав которого входят оксиды кальция, железа, магния, диоксид кремния, глинозем. Цемент, затворенный водой, дает сильную усадку при твердении, что приводит к появлению трещин в отвердевшем продукте. Поэтому вяжущее используется в сочетании с заполнителями и наполнителями, предотвращающими растрескивание цементного камня. Наиболее популярным мелким заполнителем является песок.

Назначение строительных цементных растворов

Цементно-песчаные (и другие) строительные растворы регламентируются ГОСТом 28013-98. По назначению их разделяют на следующие виды:

  • Кладочные, в том числе для монтажных работ. Используются при ведении кирпичной или каменной кладки, для заполнения швов между бетонными и железобетонными панелями, выравнивания полов, заливки площадок, не предназначенных для восприятия серьезных нагрузок. Кладочный состав обеспечивает повышение тепло- и звукоизоляционных характеристик строения.

  • Штукатурные. Применяются для выравнивания основания и защиты стенового материала от воздействия атмосферных явлений.
  • Облицовочные. Служат для облицовки вертикальных и горизонтальных строительных конструкций керамической и керамогранитной плиткой.

В зависимости от назначения цементного раствора в его составе может быть песок различных фракций.







Назначение раствораКрупность зерен песка, мм
Для кладки, кроме бутового камня2,5
Для бутовой кладки5,0
Для штукатурки, кроме накрывочного слоя2,5
Для накрывочного штукатурного слоя1,25
Для облицовочного слоя1,25

В соответствии с ГОСТом 28013-98 цементные растворы различаются по маркам прочности на сжатие.

Таблица областей применения в зависимости от марки прочности цементного раствора







Марки по прочности на сжатиеОбласти применения
М50Заделка щелей внутри помещений
М75Внутренние кладочные работы
М100Наружная кладка кирпича и блоков, устройство стяжки пола
М150Заполнение швов в конструкциях из тяжелых бетонов, изготовления стяжки, при оборудовании гидротехнических объектов
М200Благодаря высокой водостойкости, продукт используют в качестве гидроизоляционного слоя; при изготовлении материала для конструкций, которые в процессе эксплуатации будут контактировать с агрессивными средами, используется сульфатостойкий цемент

Компоненты строительных цементных растворов

Для получения качественного строительного материала каждый компонент должен соответствовать требованиям нормативов:

  • Цемент. В общих случаях используется портландцемент марок 400 и 500 без минеральных добавок или с минеральными добавками в количестве до 20%. Для особых эксплуатационных условий применяют сульфатостойкое, гидротехническое, пластифицированное цементное вяжущее.
  • Песок. Должен соответствовать ГОСТу 8736-2014 «Песок для строительных работ». Для изготовления растворов используют речной и карьерный песок, очищенный от илистых и глинистых включений, снижающих качество готового продукта.
  • Вода. Из питьевого трубопровода или проверенная на качество в лабораторных условиях. Температура +15…+20°C.

Цемент, песок и вода – основные компоненты строительного цементного раствора, но также в рецепт включаются добавки, придающие пластичной смеси или конечному продукту определенные свойства:

  • Эластификаторы. Улучшают эластичность раствора и его адгезию к основанию, повышают устойчивость затвердевшего продукта к появлению трещин и влагостойкость. Функции эластификатора может выполнять ПВА.
  • Пластификаторы и их более мощный вариант – суперпластификаторы. Увеличивают подвижность пластичной смеси, сокращают расход материала, уменьшают его склонность к расслоению. Наиболее простой вариант – применение моющих растворов. Их добавляют не в сухую смесь, а в воду.
  • Гидроизоляторы. Такие добавки ускоряют схватывание и твердение раствора, повышают водонепроницаемость готового слоя.
  • Латексные добавки. Сообщают готовому продукту широкий спектр полезных свойств – водостойкость, устойчивость к воздействию нефти и нефтепродуктов, других агрессивных химических веществ.
  • Противоморозные. Применяются при ведении работ в холодный период года.
  • Сажа, графит и другие красящие вещества. Не влияют на физические характеристики материала, применяются только для изменения цвета готового продукта.

Удельный вес цементно-песчаного раствора зависит от вида и пропорций составляющих, в среднем он равен 1800 кг/м3.

Этапы приготовления

Пропорции компонентов зависят от области применения цементного раствора, а следовательно, от марки прочности на сжатие.

Таблица пропорций компонентов раствора – цемента и песка по массе











Марка раствораМарка цементаПропорции компонентов
ЦементПесок
М50М40017,4
М75М40015,4
  М100М40014,3
М50014,3
  М150М40013,25
М50013,9
  М200М40012,5
М50013

При небольших объемах работ приготовление цементно-песчаного раствора возможно вручную.

Последовательность:

  • Смешивают вяжущее и песок в сухом состоянии в металлической емкости или на стальном листе. Делать это на грунте не рекомендуется, поскольку состав загрязняется.
  • После того как смесь приобретет однородный сероватый цвет ее сгребают в гряду или кучку, на вершине которой делают небольшое углубление. В него небольшими порциями добавляют воду.
  • Полученный состав вымешивают.

 

Готовый продукт должен напоминать по густоте сметану, след от лопаты должен быть четким, не расплывчатым.

Приготовление материала в бетономешалке обеспечивает высокую скорость процесса и хорошее качество готовой пластичной смеси, благодаря тщательному перемешиванию и получению полностью однородного продукта.

Как сделать цементный раствор в бетономешалке:

  • В барабан заливают примерно половину положенного объема воды. Примерное количество воды – половина от объема цемента.
  • Перед тем как развести цемент, в воду вводят добавки, например моющее средство, которое должно полностью раствориться с образованием равномерной пены. Время перемешивания – 3-5 минут.
  • В барабан добавляют цемент и половину песка. Время перемешивания – 1-3 минуты.
  • Вводят остаток песка, перемешивают, регулируют плотность цементного раствора путем введения нужного количества воды.
  • Последний замес – 3-5 минут.

В результате получается однородный продукт, без комков, воздушных пузырей и расслоений. Приготовленный пластичный материал не должен растекаться и рассыпаться. Для проверки его готовности комок выкладывают на ровную поверхность. Требования – из комка не должна вытекать вода, со временем он немного оседает без потери первоначальной формы.

Цементный раствор — пропорции: как развести и сколько песка и цемента в 1 м3, соотношение частей и расход

Цемент – это основной строительный материал, который используется практически во всех отраслях народного хозяйства. С помощью данного вещества можно получить очень прочные продукты, способные выдерживать высокие нагрузки и противостоять внешним воздействиям. Но все эти характеристики зависят также и от используемых компонентов, и технологии приготовления. Цементные растворы широко применяются в строительстве, так как позволяют упростить много операций.

Особенности

Цементные растворы представляют собой искусственные смеси, которые после застывания образуют прочную структуру. Состоит подобный продукт из нескольких основных компонентов.

  • Песок. Он используется в качестве основного компонента, так как объединяет в себе мелкую структуру и относительно высокую прочность. Для приготовления растворов могут использовать речной или карьерный песок. Первый тип материала применяется при монолитном строительстве, позволяя получить очень прочные продукты.
  • Вода. Данный компонент нужен для связывания песка и цемента. Количество жидкости подбирается в зависимости от марки и предназначения раствора.
  • Цемент. Это основное вещество, которое отличается высокой адгезией с другими материалами. Сегодня существует несколько марок цемента, предназначенного для эксплуатации в различных условиях. Отличаются они показателями прочности.
  • Пластификаторы. Технически это различные виды примесей, которые предназначаются для изменения физических или химических свойств раствора. Они используются не так часто, так как это может значительно увеличивать стоимость продукта.

Подобную продукцию используют для решения следующих видов задач:

  • оштукатуривание – некоторыми растворами покрывают стены для защиты строительного материала, а также с целью выравнивания основания;
  • кладка – цементные смеси прекрасно связывают между собой кирпич или газоблок, поэтому их используют в качестве своеобразного клея, располагающегося внутри каждого шва;
  • создание железобетонных конструкций.

Виды составов и требования

Основной характеристикой цементного раствора является его прочность. Она обусловлена соотношением цемента и песка. Состав продукта может изменяться штучно, что позволяет получить несколько видов смесей. Каждый из них предназначается для использования в определенных условиях. Поэтому важно правильно готовить продукцию при строительстве различных объектов.

Типы

Одним из критериев разделения цементных смесей на виды являются пропорции внутренних компонентов. Стоит обратить внимание, что в одном составе может присутствовать только одна марка цемента. Но они могут также изменяться, так как прочность будет зависеть уже только от концентрации компонентов. Условно их разделяют на несколько марок.

  • М100 (М150) – эти смеси отличаются незначительной прочностью. Для их приготовления можно использовать цемент марок М200–М500. Но при этом необходимо правильно подбирать пропорции цементно-песчаных компонентов.
  • М200 – это один из самых распространенных видов растворов. Его используют очень часто в быту для строительства дорожек и или формирования покрытий, которые не поддаются значительным нагрузкам. Сохнет данная смесь относительно быстро, но при этом требует соблюдения определенных микроклиматических условий.
  • М300 – данный вид раствора можно уже отнести к бетонным типам. Он используется для приготовления бетонов, из которых затем изготавливают прочные плиты перекрытия, заливают фундаменты и много другого.
  • М400 – это прочный бетон, который состоит из качественных марок цемента (М350, М400, М500). Используют его в строительстве фундаментов для многоэтажных домов. Данный раствор составляет основу для изготовления железобетонных плит перекрытия и других подобных изделий.
  • М500 – это самый прочный бетон, который способен выдерживать очень высокие нагрузки. Он сохраняет свои первоначальные свойства на протяжении многих лет и при воздействии различных раздражителей.

Добавки

Качество цементного раствора зависит практически от всех его составляющих, которые присутствуют внутри. Иногда свойств песчано-цементной смеси недостаточно, поэтому нужно адаптировать их под определенные условия.

Решается эта проблема с помощью добавления в состав различных примесей. С помощью подобных присадок получают так называемое жидкое стекло. Эти продукты используются для оштукатуривания стен и других поверхностей.

Сегодня в качестве добавок для цементных растворов используют несколько продуктов.

  • Известь. В качестве добавок применяют только гашенные ее виды. Внедрение этого вещества позволяет немного увеличить паропроницаемость и прочность. Но чтобы приготовить подобную продукцию, следует соблюдать точные пропорции. Очень часто на основе извести изготавливают штукатурки, которые прекрасно наносятся на стены.
  • ПВА. Клей улучшает адгезию и пластичность раствора. Важно правильно подбирать концентрацию добавки, чтобы получить хорошую смесь.
  • Моющие средства. Подобные продукты влияют на пластичность раствора. Добавляют их в состав только после воды. Здесь также обязательно соблюдается точная доза примеси на единицу объема.
  • Сажа или графит. Эти вещества практически не влияют на физические свойства смеси. Используются они только в качестве красителей для изменения цвета готового продукта.

Соотношение песка и цемента

Приготовить цементно-песчаный раствор можно даже в домашних условиях, так как состоит он из доступных компонентов. Приобрести их довольно легко практически в любом строительном магазине. Но отличаются растворы соотношением цемента и песка, от которых и зависит расход и физические характеристики материала.

Кирпичная кладка

Скрепление кирпичей – это одна из основных задач цементных растворов. Для таких целей используют не особо прочные марки (до М400). Для получения подобной смеси специалисты рекомендуют использовать песок средней фракции с минимальным уровнем влажности. Приготовить кладочный раствор можно используя различные марки цемента. Но при этом будет уже изменяться соотношение цемента и песка. Некоторые пропорции представлены в таблице 1.

Таблица 1. Соотношения компонентов в зависимости от марки цемента

Обратите внимание, что расчет желательно проводить согласно только одних единиц измерения. В большинстве случаев все части рассчитывают на 1 м³. Но при этом массам различных материалов в кубе может отличаться.

Приготовление бетона

Бетонные конструкции также очень часто используются в современной промышленности. Эти материалы изготавливаются на заводах или непосредственно на строительных площадках. Прочность таких изделий также зависит от цемента, который планируется использовать. Технически бетон можно изготовить и из раствора марки М100, но он не будет выдерживать нагрузки, и отличаться минимальным сроком службы.

Еще одной особенностью бетонов является наличие в составе щебня и других вспомогательных компонентов. Они внедряются с целью изменения технических характеристик продукта.

Следует отметить, что смешиваться они могут в различных комбинациях, что зависит от среды использования бетона.

Сегодня многие специалисты используют такое соотношение компонентов бетонных растворов, как:

  • 4 части щебня;
  • 1 часть цемента;
  • 2 части песка;
  • ½ части воды.

Обратите внимание, что пропорции могут изменяться, если еще планируется использовать различные полимерные добавки. В таких случаях желательно обращать внимание на рекомендации производителей данных примесей.

Для штукатурки и стяжки

Заливка пола очень часто предполагает использование относительно жидких цементных растворов. Такая консистенция позволяет равномерно распределить смесь на основании и получить горизонтальную поверхность. Штукатурка же практически всегда состоит только из чистого песка, цемента и воды. Ее густота может быть разной, так как все зависит от того, где ее планируется использовать.

Самой распространенной пропорцией для получения штукатурных смесей является отношение цемента к песку 1: 5. Консистенция адаптируется под потребности мастера.

Особое внимание следует уделить стяжкам, которые поддаются значительным и постоянным нагрузкам. Для таких поверхностей следует использовать материалы, у которых пороговая прочность не меньше 10 МПа. Достигается это за счет использования бетонов марки не ниже М150. Пропорция приготовления раствора для стяжки зависит от следующих факторов:

  • использование смесей с целью сокрытия различных коммуникационных элементов;
  • толщина выравнивания поверхности. Если нужно просто укрепить пол с небольшими перепадами, тогда применяют более жидкие составы. Для более толстых слоев желательно использовать прочные виды растворов.

Таблица 2. Пропорции песка и цемента в стяжках

Обратите внимание, что пропорции компонентов в большинстве случаев повторяются. Но при этом прочность получаемого раствора на выходе отличается. Это важно учитывать, если продукция будет применяться в специфических условиях эксплуатации.

Как правильно развести?

Процесс приготовления цементных растворов предполагает смешивание всех компонентов в определенной последовательности. Описать подобную процедуру можно несколькими последовательными шагами.

  • В первую очередь нужно определиться с типом раствора, который нужен. При этом обращают внимание на прочность результирующей смеси. Если важен этот показатель, следует провести дополнительный расчет всех составляющих. Особое внимание следует обращать на нормы или стандарты.
  • На данном этапе смешивают сухие компоненты, объем которых измеряется в 1 м³ или других подобных единицах. Перед тем как замесить раствор, следует тщательно перемешать песок и цемент, чтобы получить равномерную смесь. Поэтому так важно использовать сухие вещества.
  • Когда подготовка прошла успешно, можно разводить смесь. Для этого постепенно в нее добавляют воду и тщательно смешивают все компоненты. Лучше всего использовать бетономешалки или другие механические приспособления. Консистенция раствора регулируется с помощью жидкости.

Советы и рекомендации

Приготовление цементного раствора является простой операцией. При ее осуществлении все-таки рекомендовано соблюдать несколько простых правил, рекомендуемых производителем и опытными строителями, такие как:

  • если смесь должна быть пластичной, для этого нужно добавлять в нее жидкое мыло. Его смешивать нужно предварительно с водой;
  • следует добавлять воду небольшими порциями. Таким образом, можно контролировать густоту смеси, которая очень важна для стяжек или кладки;
  • при строительстве обязательно нужно учитывать марку самого кирпича или другого материала. Специалисты рекомендуют готовить такие смеси, которые по данным параметрам должны совпадать. Это позволит получить однородную структуру стены, которая будет отличаться прочностью;
  • для повышения теплоизоляционных характеристик штукатурок стоит добавлять в их состав перлит. При этом им нужно заменять определенную часть песка;
  • рекомендуется использовать только свежий цемент, структура которого не содержит комков. Это гарантирует высокую адгезию и равномерное смешивание.

Цементный раствор – это прекрасный материал, позволяющий получить прочные конструкции. Правильно приготовленная смесь – это залог долговечности практически любого строения и его основания.

Подробнее о пропорциях цементного раствора вы узнаете из следующего видео.

Цементный раствор — пропорции: как развести и сколько песка и цемента в 1 м3, соотношение частей и расход

Цемент – это основной строительный материал, который используется практически во всех отраслях народного хозяйства. С помощью данного вещества можно получить очень прочные продукты, способные выдерживать высокие нагрузки и противостоять внешним воздействиям. Но все эти характеристики зависят также и от используемых компонентов, и технологии приготовления. Цементные растворы широко применяются в строительстве, так как позволяют упростить много операций.

Особенности

Цементные растворы представляют собой искусственные смеси, которые после застывания образуют прочную структуру. Состоит подобный продукт из нескольких основных компонентов.

  • Песок. Он используется в качестве основного компонента, так как объединяет в себе мелкую структуру и относительно высокую прочность. Для приготовления растворов могут использовать речной или карьерный песок. Первый тип материала применяется при монолитном строительстве, позволяя получить очень прочные продукты.
  • Вода. Данный компонент нужен для связывания песка и цемента. Количество жидкости подбирается в зависимости от марки и предназначения раствора.
  • Цемент. Это основное вещество, которое отличается высокой адгезией с другими материалами. Сегодня существует несколько марок цемента, предназначенного для эксплуатации в различных условиях. Отличаются они показателями прочности.
  • Пластификаторы. Технически это различные виды примесей, которые предназначаются для изменения физических или химических свойств раствора. Они используются не так часто, так как это может значительно увеличивать стоимость продукта.

Подобную продукцию используют для решения следующих видов задач:

  • оштукатуривание – некоторыми растворами покрывают стены для защиты строительного материала, а также с целью выравнивания основания;
  • кладка – цементные смеси прекрасно связывают между собой кирпич или газоблок, поэтому их используют в качестве своеобразного клея, располагающегося внутри каждого шва;
  • создание железобетонных конструкций.

Виды составов и требования

Основной характеристикой цементного раствора является его прочность. Она обусловлена соотношением цемента и песка. Состав продукта может изменяться штучно, что позволяет получить несколько видов смесей. Каждый из них предназначается для использования в определенных условиях. Поэтому важно правильно готовить продукцию при строительстве различных объектов.

Типы

Одним из критериев разделения цементных смесей на виды являются пропорции внутренних компонентов. Стоит обратить внимание, что в одном составе может присутствовать только одна марка цемента. Но они могут также изменяться, так как прочность будет зависеть уже только от концентрации компонентов. Условно их разделяют на несколько марок.

  • М100 (М150) – эти смеси отличаются незначительной прочностью. Для их приготовления можно использовать цемент марок М200–М500. Но при этом необходимо правильно подбирать пропорции цементно-песчаных компонентов.
  • М200 – это один из самых распространенных видов растворов. Его используют очень часто в быту для строительства дорожек и или формирования покрытий, которые не поддаются значительным нагрузкам. Сохнет данная смесь относительно быстро, но при этом требует соблюдения определенных микроклиматических условий.
  • М300 – данный вид раствора можно уже отнести к бетонным типам. Он используется для приготовления бетонов, из которых затем изготавливают прочные плиты перекрытия, заливают фундаменты и много другого.
  • М400 – это прочный бетон, который состоит из качественных марок цемента (М350, М400, М500). Используют его в строительстве фундаментов для многоэтажных домов. Данный раствор составляет основу для изготовления железобетонных плит перекрытия и других подобных изделий.
  • М500 – это самый прочный бетон, который способен выдерживать очень высокие нагрузки. Он сохраняет свои первоначальные свойства на протяжении многих лет и при воздействии различных раздражителей.

Добавки

Качество цементного раствора зависит практически от всех его составляющих, которые присутствуют внутри. Иногда свойств песчано-цементной смеси недостаточно, поэтому нужно адаптировать их под определенные условия.

Решается эта проблема с помощью добавления в состав различных примесей. С помощью подобных присадок получают так называемое жидкое стекло. Эти продукты используются для оштукатуривания стен и других поверхностей.

Сегодня в качестве добавок для цементных растворов используют несколько продуктов.

  • Известь. В качестве добавок применяют только гашенные ее виды. Внедрение этого вещества позволяет немного увеличить паропроницаемость и прочность. Но чтобы приготовить подобную продукцию, следует соблюдать точные пропорции. Очень часто на основе извести изготавливают штукатурки, которые прекрасно наносятся на стены.
  • ПВА. Клей улучшает адгезию и пластичность раствора. Важно правильно подбирать концентрацию добавки, чтобы получить хорошую смесь.
  • Моющие средства. Подобные продукты влияют на пластичность раствора. Добавляют их в состав только после воды. Здесь также обязательно соблюдается точная доза примеси на единицу объема.
  • Сажа или графит. Эти вещества практически не влияют на физические свойства смеси. Используются они только в качестве красителей для изменения цвета готового продукта.

Соотношение песка и цемента

Приготовить цементно-песчаный раствор можно даже в домашних условиях, так как состоит он из доступных компонентов. Приобрести их довольно легко практически в любом строительном магазине. Но отличаются растворы соотношением цемента и песка, от которых и зависит расход и физические характеристики материала.

Кирпичная кладка

Скрепление кирпичей – это одна из основных задач цементных растворов. Для таких целей используют не особо прочные марки (до М400). Для получения подобной смеси специалисты рекомендуют использовать песок средней фракции с минимальным уровнем влажности. Приготовить кладочный раствор можно используя различные марки цемента. Но при этом будет уже изменяться соотношение цемента и песка. Некоторые пропорции представлены в таблице 1.

Таблица 1. Соотношения компонентов в зависимости от марки цемента

Обратите внимание, что расчет желательно проводить согласно только одних единиц измерения. В большинстве случаев все части рассчитывают на 1 м³. Но при этом массам различных материалов в кубе может отличаться.

Приготовление бетона

Бетонные конструкции также очень часто используются в современной промышленности. Эти материалы изготавливаются на заводах или непосредственно на строительных площадках. Прочность таких изделий также зависит от цемента, который планируется использовать. Технически бетон можно изготовить и из раствора марки М100, но он не будет выдерживать нагрузки, и отличаться минимальным сроком службы.

Еще одной особенностью бетонов является наличие в составе щебня и других вспомогательных компонентов. Они внедряются с целью изменения технических характеристик продукта.

Следует отметить, что смешиваться они могут в различных комбинациях, что зависит от среды использования бетона.

Сегодня многие специалисты используют такое соотношение компонентов бетонных растворов, как:

  • 4 части щебня;
  • 1 часть цемента;
  • 2 части песка;
  • ½ части воды.

Обратите внимание, что пропорции могут изменяться, если еще планируется использовать различные полимерные добавки. В таких случаях желательно обращать внимание на рекомендации производителей данных примесей.

Для штукатурки и стяжки

Заливка пола очень часто предполагает использование относительно жидких цементных растворов. Такая консистенция позволяет равномерно распределить смесь на основании и получить горизонтальную поверхность. Штукатурка же практически всегда состоит только из чистого песка, цемента и воды. Ее густота может быть разной, так как все зависит от того, где ее планируется использовать.

Самой распространенной пропорцией для получения штукатурных смесей является отношение цемента к песку 1: 5. Консистенция адаптируется под потребности мастера.

Особое внимание следует уделить стяжкам, которые поддаются значительным и постоянным нагрузкам. Для таких поверхностей следует использовать материалы, у которых пороговая прочность не меньше 10 МПа. Достигается это за счет использования бетонов марки не ниже М150. Пропорция приготовления раствора для стяжки зависит от следующих факторов:

  • использование смесей с целью сокрытия различных коммуникационных элементов;
  • толщина выравнивания поверхности. Если нужно просто укрепить пол с небольшими перепадами, тогда применяют более жидкие составы. Для более толстых слоев желательно использовать прочные виды растворов.

Таблица 2. Пропорции песка и цемента в стяжках

Обратите внимание, что пропорции компонентов в большинстве случаев повторяются. Но при этом прочность получаемого раствора на выходе отличается. Это важно учитывать, если продукция будет применяться в специфических условиях эксплуатации.

Как правильно развести?

Процесс приготовления цементных растворов предполагает смешивание всех компонентов в определенной последовательности. Описать подобную процедуру можно несколькими последовательными шагами.

  • В первую очередь нужно определиться с типом раствора, который нужен. При этом обращают внимание на прочность результирующей смеси. Если важен этот показатель, следует провести дополнительный расчет всех составляющих. Особое внимание следует обращать на нормы или стандарты.
  • На данном этапе смешивают сухие компоненты, объем которых измеряется в 1 м³ или других подобных единицах. Перед тем как замесить раствор, следует тщательно перемешать песок и цемент, чтобы получить равномерную смесь. Поэтому так важно использовать сухие вещества.
  • Когда подготовка прошла успешно, можно разводить смесь. Для этого постепенно в нее добавляют воду и тщательно смешивают все компоненты. Лучше всего использовать бетономешалки или другие механические приспособления. Консистенция раствора регулируется с помощью жидкости.

Советы и рекомендации

Приготовление цементного раствора является простой операцией. При ее осуществлении все-таки рекомендовано соблюдать несколько простых правил, рекомендуемых производителем и опытными строителями, такие как:

  • если смесь должна быть пластичной, для этого нужно добавлять в нее жидкое мыло. Его смешивать нужно предварительно с водой;
  • следует добавлять воду небольшими порциями. Таким образом, можно контролировать густоту смеси, которая очень важна для стяжек или кладки;
  • при строительстве обязательно нужно учитывать марку самого кирпича или другого материала. Специалисты рекомендуют готовить такие смеси, которые по данным параметрам должны совпадать. Это позволит получить однородную структуру стены, которая будет отличаться прочностью;
  • для повышения теплоизоляционных характеристик штукатурок стоит добавлять в их состав перлит. При этом им нужно заменять определенную часть песка;
  • рекомендуется использовать только свежий цемент, структура которого не содержит комков. Это гарантирует высокую адгезию и равномерное смешивание.

Цементный раствор – это прекрасный материал, позволяющий получить прочные конструкции. Правильно приготовленная смесь – это залог долговечности практически любого строения и его основания.

Подробнее о пропорциях цементного раствора вы узнаете из следующего видео.

Цементный раствор — пропорции: как развести и сколько песка и цемента в 1 м3, соотношение частей и расход

Цемент – это основной строительный материал, который используется практически во всех отраслях народного хозяйства. С помощью данного вещества можно получить очень прочные продукты, способные выдерживать высокие нагрузки и противостоять внешним воздействиям. Но все эти характеристики зависят также и от используемых компонентов, и технологии приготовления. Цементные растворы широко применяются в строительстве, так как позволяют упростить много операций.

Особенности

Цементные растворы представляют собой искусственные смеси, которые после застывания образуют прочную структуру. Состоит подобный продукт из нескольких основных компонентов.

  • Песок. Он используется в качестве основного компонента, так как объединяет в себе мелкую структуру и относительно высокую прочность. Для приготовления растворов могут использовать речной или карьерный песок. Первый тип материала применяется при монолитном строительстве, позволяя получить очень прочные продукты.
  • Вода. Данный компонент нужен для связывания песка и цемента. Количество жидкости подбирается в зависимости от марки и предназначения раствора.
  • Цемент. Это основное вещество, которое отличается высокой адгезией с другими материалами. Сегодня существует несколько марок цемента, предназначенного для эксплуатации в различных условиях. Отличаются они показателями прочности.
  • Пластификаторы. Технически это различные виды примесей, которые предназначаются для изменения физических или химических свойств раствора. Они используются не так часто, так как это может значительно увеличивать стоимость продукта.

Подобную продукцию используют для решения следующих видов задач:

  • оштукатуривание – некоторыми растворами покрывают стены для защиты строительного материала, а также с целью выравнивания основания;
  • кладка – цементные смеси прекрасно связывают между собой кирпич или газоблок, поэтому их используют в качестве своеобразного клея, располагающегося внутри каждого шва;
  • создание железобетонных конструкций.

Виды составов и требования

Основной характеристикой цементного раствора является его прочность. Она обусловлена соотношением цемента и песка. Состав продукта может изменяться штучно, что позволяет получить несколько видов смесей. Каждый из них предназначается для использования в определенных условиях. Поэтому важно правильно готовить продукцию при строительстве различных объектов.

Типы

Одним из критериев разделения цементных смесей на виды являются пропорции внутренних компонентов. Стоит обратить внимание, что в одном составе может присутствовать только одна марка цемента. Но они могут также изменяться, так как прочность будет зависеть уже только от концентрации компонентов. Условно их разделяют на несколько марок.

  • М100 (М150) – эти смеси отличаются незначительной прочностью. Для их приготовления можно использовать цемент марок М200–М500. Но при этом необходимо правильно подбирать пропорции цементно-песчаных компонентов.
  • М200 – это один из самых распространенных видов растворов. Его используют очень часто в быту для строительства дорожек и или формирования покрытий, которые не поддаются значительным нагрузкам. Сохнет данная смесь относительно быстро, но при этом требует соблюдения определенных микроклиматических условий.
  • М300 – данный вид раствора можно уже отнести к бетонным типам. Он используется для приготовления бетонов, из которых затем изготавливают прочные плиты перекрытия, заливают фундаменты и много другого.
  • М400 – это прочный бетон, который состоит из качественных марок цемента (М350, М400, М500). Используют его в строительстве фундаментов для многоэтажных домов. Данный раствор составляет основу для изготовления железобетонных плит перекрытия и других подобных изделий.
  • М500 – это самый прочный бетон, который способен выдерживать очень высокие нагрузки. Он сохраняет свои первоначальные свойства на протяжении многих лет и при воздействии различных раздражителей.

Добавки

Качество цементного раствора зависит практически от всех его составляющих, которые присутствуют внутри. Иногда свойств песчано-цементной смеси недостаточно, поэтому нужно адаптировать их под определенные условия.

Решается эта проблема с помощью добавления в состав различных примесей. С помощью подобных присадок получают так называемое жидкое стекло. Эти продукты используются для оштукатуривания стен и других поверхностей.

Сегодня в качестве добавок для цементных растворов используют несколько продуктов.

  • Известь. В качестве добавок применяют только гашенные ее виды. Внедрение этого вещества позволяет немного увеличить паропроницаемость и прочность. Но чтобы приготовить подобную продукцию, следует соблюдать точные пропорции. Очень часто на основе извести изготавливают штукатурки, которые прекрасно наносятся на стены.
  • ПВА. Клей улучшает адгезию и пластичность раствора. Важно правильно подбирать концентрацию добавки, чтобы получить хорошую смесь.
  • Моющие средства. Подобные продукты влияют на пластичность раствора. Добавляют их в состав только после воды. Здесь также обязательно соблюдается точная доза примеси на единицу объема.
  • Сажа или графит. Эти вещества практически не влияют на физические свойства смеси. Используются они только в качестве красителей для изменения цвета готового продукта.

Соотношение песка и цемента

Приготовить цементно-песчаный раствор можно даже в домашних условиях, так как состоит он из доступных компонентов. Приобрести их довольно легко практически в любом строительном магазине. Но отличаются растворы соотношением цемента и песка, от которых и зависит расход и физические характеристики материала.

Кирпичная кладка

Скрепление кирпичей – это одна из основных задач цементных растворов. Для таких целей используют не особо прочные марки (до М400). Для получения подобной смеси специалисты рекомендуют использовать песок средней фракции с минимальным уровнем влажности. Приготовить кладочный раствор можно используя различные марки цемента. Но при этом будет уже изменяться соотношение цемента и песка. Некоторые пропорции представлены в таблице 1.

Таблица 1. Соотношения компонентов в зависимости от марки цемента

Обратите внимание, что расчет желательно проводить согласно только одних единиц измерения. В большинстве случаев все части рассчитывают на 1 м³. Но при этом массам различных материалов в кубе может отличаться.

Приготовление бетона

Бетонные конструкции также очень часто используются в современной промышленности. Эти материалы изготавливаются на заводах или непосредственно на строительных площадках. Прочность таких изделий также зависит от цемента, который планируется использовать. Технически бетон можно изготовить и из раствора марки М100, но он не будет выдерживать нагрузки, и отличаться минимальным сроком службы.

Еще одной особенностью бетонов является наличие в составе щебня и других вспомогательных компонентов. Они внедряются с целью изменения технических характеристик продукта.

Следует отметить, что смешиваться они могут в различных комбинациях, что зависит от среды использования бетона.

Сегодня многие специалисты используют такое соотношение компонентов бетонных растворов, как:

  • 4 части щебня;
  • 1 часть цемента;
  • 2 части песка;
  • ½ части воды.

Обратите внимание, что пропорции могут изменяться, если еще планируется использовать различные полимерные добавки. В таких случаях желательно обращать внимание на рекомендации производителей данных примесей.

Для штукатурки и стяжки

Заливка пола очень часто предполагает использование относительно жидких цементных растворов. Такая консистенция позволяет равномерно распределить смесь на основании и получить горизонтальную поверхность. Штукатурка же практически всегда состоит только из чистого песка, цемента и воды. Ее густота может быть разной, так как все зависит от того, где ее планируется использовать.

Самой распространенной пропорцией для получения штукатурных смесей является отношение цемента к песку 1: 5. Консистенция адаптируется под потребности мастера.

Особое внимание следует уделить стяжкам, которые поддаются значительным и постоянным нагрузкам. Для таких поверхностей следует использовать материалы, у которых пороговая прочность не меньше 10 МПа. Достигается это за счет использования бетонов марки не ниже М150. Пропорция приготовления раствора для стяжки зависит от следующих факторов:

  • использование смесей с целью сокрытия различных коммуникационных элементов;
  • толщина выравнивания поверхности. Если нужно просто укрепить пол с небольшими перепадами, тогда применяют более жидкие составы. Для более толстых слоев желательно использовать прочные виды растворов.

Таблица 2. Пропорции песка и цемента в стяжках

Обратите внимание, что пропорции компонентов в большинстве случаев повторяются. Но при этом прочность получаемого раствора на выходе отличается. Это важно учитывать, если продукция будет применяться в специфических условиях эксплуатации.

Как правильно развести?

Процесс приготовления цементных растворов предполагает смешивание всех компонентов в определенной последовательности. Описать подобную процедуру можно несколькими последовательными шагами.

  • В первую очередь нужно определиться с типом раствора, который нужен. При этом обращают внимание на прочность результирующей смеси. Если важен этот показатель, следует провести дополнительный расчет всех составляющих. Особое внимание следует обращать на нормы или стандарты.
  • На данном этапе смешивают сухие компоненты, объем которых измеряется в 1 м³ или других подобных единицах. Перед тем как замесить раствор, следует тщательно перемешать песок и цемент, чтобы получить равномерную смесь. Поэтому так важно использовать сухие вещества.
  • Когда подготовка прошла успешно, можно разводить смесь. Для этого постепенно в нее добавляют воду и тщательно смешивают все компоненты. Лучше всего использовать бетономешалки или другие механические приспособления. Консистенция раствора регулируется с помощью жидкости.

Советы и рекомендации

Приготовление цементного раствора является простой операцией. При ее осуществлении все-таки рекомендовано соблюдать несколько простых правил, рекомендуемых производителем и опытными строителями, такие как:

  • если смесь должна быть пластичной, для этого нужно добавлять в нее жидкое мыло. Его смешивать нужно предварительно с водой;
  • следует добавлять воду небольшими порциями. Таким образом, можно контролировать густоту смеси, которая очень важна для стяжек или кладки;
  • при строительстве обязательно нужно учитывать марку самого кирпича или другого материала. Специалисты рекомендуют готовить такие смеси, которые по данным параметрам должны совпадать. Это позволит получить однородную структуру стены, которая будет отличаться прочностью;
  • для повышения теплоизоляционных характеристик штукатурок стоит добавлять в их состав перлит. При этом им нужно заменять определенную часть песка;
  • рекомендуется использовать только свежий цемент, структура которого не содержит комков. Это гарантирует высокую адгезию и равномерное смешивание.

Цементный раствор – это прекрасный материал, позволяющий получить прочные конструкции. Правильно приготовленная смесь – это залог долговечности практически любого строения и его основания.

Подробнее о пропорциях цементного раствора вы узнаете из следующего видео.

Цементный раствор — пропорции: как развести и сколько песка и цемента в 1 м3, соотношение частей и расход

Цемент – это основной строительный материал, который используется практически во всех отраслях народного хозяйства. С помощью данного вещества можно получить очень прочные продукты, способные выдерживать высокие нагрузки и противостоять внешним воздействиям. Но все эти характеристики зависят также и от используемых компонентов, и технологии приготовления. Цементные растворы широко применяются в строительстве, так как позволяют упростить много операций.

Особенности

Цементные растворы представляют собой искусственные смеси, которые после застывания образуют прочную структуру. Состоит подобный продукт из нескольких основных компонентов.

  • Песок. Он используется в качестве основного компонента, так как объединяет в себе мелкую структуру и относительно высокую прочность. Для приготовления растворов могут использовать речной или карьерный песок. Первый тип материала применяется при монолитном строительстве, позволяя получить очень прочные продукты.
  • Вода. Данный компонент нужен для связывания песка и цемента. Количество жидкости подбирается в зависимости от марки и предназначения раствора.
  • Цемент. Это основное вещество, которое отличается высокой адгезией с другими материалами. Сегодня существует несколько марок цемента, предназначенного для эксплуатации в различных условиях. Отличаются они показателями прочности.
  • Пластификаторы. Технически это различные виды примесей, которые предназначаются для изменения физических или химических свойств раствора. Они используются не так часто, так как это может значительно увеличивать стоимость продукта.

Подобную продукцию используют для решения следующих видов задач:

  • оштукатуривание – некоторыми растворами покрывают стены для защиты строительного материала, а также с целью выравнивания основания;
  • кладка – цементные смеси прекрасно связывают между собой кирпич или газоблок, поэтому их используют в качестве своеобразного клея, располагающегося внутри каждого шва;
  • создание железобетонных конструкций.

Виды составов и требования

Основной характеристикой цементного раствора является его прочность. Она обусловлена соотношением цемента и песка. Состав продукта может изменяться штучно, что позволяет получить несколько видов смесей. Каждый из них предназначается для использования в определенных условиях. Поэтому важно правильно готовить продукцию при строительстве различных объектов.

Типы

Одним из критериев разделения цементных смесей на виды являются пропорции внутренних компонентов. Стоит обратить внимание, что в одном составе может присутствовать только одна марка цемента. Но они могут также изменяться, так как прочность будет зависеть уже только от концентрации компонентов. Условно их разделяют на несколько марок.

  • М100 (М150) – эти смеси отличаются незначительной прочностью. Для их приготовления можно использовать цемент марок М200–М500. Но при этом необходимо правильно подбирать пропорции цементно-песчаных компонентов.
  • М200 – это один из самых распространенных видов растворов. Его используют очень часто в быту для строительства дорожек и или формирования покрытий, которые не поддаются значительным нагрузкам. Сохнет данная смесь относительно быстро, но при этом требует соблюдения определенных микроклиматических условий.
  • М300 – данный вид раствора можно уже отнести к бетонным типам. Он используется для приготовления бетонов, из которых затем изготавливают прочные плиты перекрытия, заливают фундаменты и много другого.
  • М400 – это прочный бетон, который состоит из качественных марок цемента (М350, М400, М500). Используют его в строительстве фундаментов для многоэтажных домов. Данный раствор составляет основу для изготовления железобетонных плит перекрытия и других подобных изделий.
  • М500 – это самый прочный бетон, который способен выдерживать очень высокие нагрузки. Он сохраняет свои первоначальные свойства на протяжении многих лет и при воздействии различных раздражителей.

Добавки

Качество цементного раствора зависит практически от всех его составляющих, которые присутствуют внутри. Иногда свойств песчано-цементной смеси недостаточно, поэтому нужно адаптировать их под определенные условия.

Решается эта проблема с помощью добавления в состав различных примесей. С помощью подобных присадок получают так называемое жидкое стекло. Эти продукты используются для оштукатуривания стен и других поверхностей.

Сегодня в качестве добавок для цементных растворов используют несколько продуктов.

  • Известь. В качестве добавок применяют только гашенные ее виды. Внедрение этого вещества позволяет немного увеличить паропроницаемость и прочность. Но чтобы приготовить подобную продукцию, следует соблюдать точные пропорции. Очень часто на основе извести изготавливают штукатурки, которые прекрасно наносятся на стены.
  • ПВА. Клей улучшает адгезию и пластичность раствора. Важно правильно подбирать концентрацию добавки, чтобы получить хорошую смесь.
  • Моющие средства. Подобные продукты влияют на пластичность раствора. Добавляют их в состав только после воды. Здесь также обязательно соблюдается точная доза примеси на единицу объема.
  • Сажа или графит. Эти вещества практически не влияют на физические свойства смеси. Используются они только в качестве красителей для изменения цвета готового продукта.

Соотношение песка и цемента

Приготовить цементно-песчаный раствор можно даже в домашних условиях, так как состоит он из доступных компонентов. Приобрести их довольно легко практически в любом строительном магазине. Но отличаются растворы соотношением цемента и песка, от которых и зависит расход и физические характеристики материала.

Кирпичная кладка

Скрепление кирпичей – это одна из основных задач цементных растворов. Для таких целей используют не особо прочные марки (до М400). Для получения подобной смеси специалисты рекомендуют использовать песок средней фракции с минимальным уровнем влажности. Приготовить кладочный раствор можно используя различные марки цемента. Но при этом будет уже изменяться соотношение цемента и песка. Некоторые пропорции представлены в таблице 1.

Таблица 1. Соотношения компонентов в зависимости от марки цемента

Обратите внимание, что расчет желательно проводить согласно только одних единиц измерения. В большинстве случаев все части рассчитывают на 1 м³. Но при этом массам различных материалов в кубе может отличаться.

Приготовление бетона

Бетонные конструкции также очень часто используются в современной промышленности. Эти материалы изготавливаются на заводах или непосредственно на строительных площадках. Прочность таких изделий также зависит от цемента, который планируется использовать. Технически бетон можно изготовить и из раствора марки М100, но он не будет выдерживать нагрузки, и отличаться минимальным сроком службы.

Еще одной особенностью бетонов является наличие в составе щебня и других вспомогательных компонентов. Они внедряются с целью изменения технических характеристик продукта.

Следует отметить, что смешиваться они могут в различных комбинациях, что зависит от среды использования бетона.

Сегодня многие специалисты используют такое соотношение компонентов бетонных растворов, как:

  • 4 части щебня;
  • 1 часть цемента;
  • 2 части песка;
  • ½ части воды.

Обратите внимание, что пропорции могут изменяться, если еще планируется использовать различные полимерные добавки. В таких случаях желательно обращать внимание на рекомендации производителей данных примесей.

Для штукатурки и стяжки

Заливка пола очень часто предполагает использование относительно жидких цементных растворов. Такая консистенция позволяет равномерно распределить смесь на основании и получить горизонтальную поверхность. Штукатурка же практически всегда состоит только из чистого песка, цемента и воды. Ее густота может быть разной, так как все зависит от того, где ее планируется использовать.

Самой распространенной пропорцией для получения штукатурных смесей является отношение цемента к песку 1: 5. Консистенция адаптируется под потребности мастера.

Особое внимание следует уделить стяжкам, которые поддаются значительным и постоянным нагрузкам. Для таких поверхностей следует использовать материалы, у которых пороговая прочность не меньше 10 МПа. Достигается это за счет использования бетонов марки не ниже М150. Пропорция приготовления раствора для стяжки зависит от следующих факторов:

  • использование смесей с целью сокрытия различных коммуникационных элементов;
  • толщина выравнивания поверхности. Если нужно просто укрепить пол с небольшими перепадами, тогда применяют более жидкие составы. Для более толстых слоев желательно использовать прочные виды растворов.

Таблица 2. Пропорции песка и цемента в стяжках

Обратите внимание, что пропорции компонентов в большинстве случаев повторяются. Но при этом прочность получаемого раствора на выходе отличается. Это важно учитывать, если продукция будет применяться в специфических условиях эксплуатации.

Как правильно развести?

Процесс приготовления цементных растворов предполагает смешивание всех компонентов в определенной последовательности. Описать подобную процедуру можно несколькими последовательными шагами.

  • В первую очередь нужно определиться с типом раствора, который нужен. При этом обращают внимание на прочность результирующей смеси. Если важен этот показатель, следует провести дополнительный расчет всех составляющих. Особое внимание следует обращать на нормы или стандарты.
  • На данном этапе смешивают сухие компоненты, объем которых измеряется в 1 м³ или других подобных единицах. Перед тем как замесить раствор, следует тщательно перемешать песок и цемент, чтобы получить равномерную смесь. Поэтому так важно использовать сухие вещества.
  • Когда подготовка прошла успешно, можно разводить смесь. Для этого постепенно в нее добавляют воду и тщательно смешивают все компоненты. Лучше всего использовать бетономешалки или другие механические приспособления. Консистенция раствора регулируется с помощью жидкости.

Советы и рекомендации

Приготовление цементного раствора является простой операцией. При ее осуществлении все-таки рекомендовано соблюдать несколько простых правил, рекомендуемых производителем и опытными строителями, такие как:

  • если смесь должна быть пластичной, для этого нужно добавлять в нее жидкое мыло. Его смешивать нужно предварительно с водой;
  • следует добавлять воду небольшими порциями. Таким образом, можно контролировать густоту смеси, которая очень важна для стяжек или кладки;
  • при строительстве обязательно нужно учитывать марку самого кирпича или другого материала. Специалисты рекомендуют готовить такие смеси, которые по данным параметрам должны совпадать. Это позволит получить однородную структуру стены, которая будет отличаться прочностью;
  • для повышения теплоизоляционных характеристик штукатурок стоит добавлять в их состав перлит. При этом им нужно заменять определенную часть песка;
  • рекомендуется использовать только свежий цемент, структура которого не содержит комков. Это гарантирует высокую адгезию и равномерное смешивание.

Цементный раствор – это прекрасный материал, позволяющий получить прочные конструкции. Правильно приготовленная смесь – это залог долговечности практически любого строения и его основания.

Подробнее о пропорциях цементного раствора вы узнаете из следующего видео.

Как сделать и замесить цементный раствор для фундамента, кладки кирпича и стяжки

Состав цементного раствора зависит от множества факторов, среди которых первую позицию занимает область применения данного стройматериала. Существует три основных вида работ, для которых используется раствор на основе цементного вяжущего: кладка, штукатурка и изготовление стяжки для пола. Строители знают, что штукатурные смеси должны обладать более высокой пластичностью, в сравнении с растворами для стяжки или кладочными смесями. В то же время, процесс заливки пола раствором подразумевает уменьшения срока схватывания материала и значительных показателей прочности на сжатие, чего от штукатурных растворов никто не требует.

Независимо от того, для кирпича какой формы готовится смесь, марка кладочного раствора должна быть значительно ниже марки прочности кирпича. Состав смеси будет напрямую зависеть именно от её марки. Для примера приведём 2 соотношения цемента и песка, которые покажут, как замесить раствор марок М100 и М75. При использовании цемента М400 в первом случае пропорция будет 1:4,3, во втором — 1:5,5. Количество воды определяется экспериментально, готовый раствор не должен падать с мастерка, его консистенция обычно напоминает густую кашу. В бетономешалке 150 л объём ингредиентов будет следующим: 16 л цемента, 88 л песка и 12 л воды, чтобы получить раствор М75, М100 потребует тех же компонентов в количестве 19 л, 84 л и 13 л соответственно.

Для тех же марок материалов составим соотношения компонентов, если стоит вопрос как сделать цементно-известковый раствор. М100 получится при использовании 1 части цемента, 0,36 частей извести и 4,3 частей песка. М75 будет иметь пропорции 1:0,56:5,5. Технология изготовления раствора аналогичная тому, как приготовить цементный раствор для фундамента, штукатурки или кладки. Сначала в ёмкости смешиваются сухие ингредиенты в объёмных пропорциях, указанных выше, затем в раствор постепенно добавляется вода в небольших количествах. Ориентироваться можно на рекомендации производителя цемента, написанные на упаковке, но окончательные соотношения можно установить только на практике.

Для кладки печи цементно-песчаный раствор без добавок никогда не используется. Здесь нужно обязательно использовать как можно более чистый песок и достаточно жирную глину в соотношении 1:1,8 – 1:2,4. Допустимое количество цемента в данном случае составляет 1 часть на 12-15 частей песчано-глиняной смеси.

Цементный раствор своими руками: виды раствора

Цементный раствор широко используется в строительстве и ремонте. Он применяется для возведения кирпичной кладки, при закладке фундамента, внутренних и внешних отделочных работах. Ни один домашний ремонт не обходится без цементного раствора – он нужен для штукатурки стен, выравнивания пола, укладки керамической плитки.

Техника изготовления раствора довольно проста – смешать компоненты может любой начинающий ремонтник. В данном материале мы разберемся, как сделать цемент своими руками с небольшим набором инструментов.

В строительстве цементный раствор применяют при закладке фундамента, возведении кирпичных стен, используют в процессе внешних и внутренних отделочных работ.

Обзор торговых марок бетона

Марка бетона – это основной показатель его прочности. Чем выше числовой номер бетона, тем надежнее и прочнее получится готовый состав.

  • M50–75 – легкий вариант цементного раствора, применяется во внутренних отделочных работах, для стяжки пола и в качестве затирки для кирпичной кладки;
  • М100–150 – так называемый “тощий бетон”, используется в качестве стяжки для пола, при изготовлении дорожек и бордюров и во вспомогательных работах при строительстве;
  • М200 – раствор для кирпичной кладки, стяжки и штукатурки при внутренних и наружных работах;
  • М300 – прочный состав, используется в основном в уличном строительстве.

Чтобы сделать цементный раствор нужной прочности, не обязательно покупать бетон соответствующей марки. В процессе приготовления смеси, бетон смешивают с песком, меняя пропорцию. Таким образом, из сырья марки М400 можно легко получить цементную смесь М100 или М200.

Цементный раствор с высокой маркой используется для наружных работ, а также при отделке особо защищенных строений. Дома использовать его не стоит, так как такой цемент много весит и обладает выраженной фактурой.

Самые распространенные марки бетона для приготовления раствора – М400 и М500. Из них можно получить цементную смесь как для отделочных, так и для небольших строительных работ.

Необходимые материалы и инструменты для работы

От состава раствора цемента зависит его прочность, технические и визуальные характеристики. Стандартный состав смеси состоит из трех элементов:

  1. сухой цемент;
  2. песок;
  3. вода.

В целях изменения качеств состава используются следующие добавки:

  • моющее средство;
  • клей ПВА;
  • известь;
  • красители.

Они влияют на цвет покрытия, помогают сделать его более пластичным и клейким.

Для того, чтобы смешать и уложить раствор цемента, понадобятся следующие инструменты:

  1. ведро или глубокий таз для размешивания, соответствующий по габаритам объему смеси;
  2. строительный миксер или дрель с насадкой;
  3. шпатель или лопатка для распределения смеси по поверхности;
  4. рукавицы и очки в целях безопасности.

Все эти инструменты наверняка найдутся в хозяйстве у людей, которые хоть раз занимались ремонтом. В противном случае, их стоит приобрести или одолжить у знакомых.

Строительный миксер или насадка на дрель помогут размешать смесь без комочков. Состав получится однородным и в нем не останутся пустоты после застывания.

Для заливки широких поверхностей рекомендуется использовать бетономешалку. Она значительно сократит время смешивания раствора и поможет рассчитать пропорции для большого объема материала.

Виды раствора

Вид цементного раствора изменяется в зависимости от марки цемента, состава компонентов и наличия добавок-пластификаторов. По составу можно выделить два вида раствора – с использованием извести и песка. Рассмотрим свойства каждого из них.

В процессе домашнего ремонта рекомендуется сначала сделать сухую смесь и только потом разводить ее водой до нужной консистенции. Влаги в составе не должно быть много – она занимает примерно половину общего объема раствора.

Цементно-известковый

Цементный раствор с добавлением извести отличается пластичностью и адгезией, поэтому чаще всего применяется в штукатурке или при укладке плитки.

В состав смеси входит цемент, известь и песок в пропорциях 1:1:6 и 1:2:9 частей соответственно. Для улучшения пластичности и клейкости раствора, в него добавляют клей ПВА, моющие средства, глинистый песок.

Если используется негашенная известь, ее следует погасить, смешав с водой. В процессе химической реакции выделяется большое количество тепла, поэтому нужно защитить руки и лицо при работе с известью.

Известь в составе позволяет высветлить цементный раствор и сделать его более пластичным. Поэтому ее часто добавляют при отделке стен внутри помещения.

Цементно-песчаный

Цементно-песчаный раствор – самый простой в изготовлении вариант покрытия. Для его замешивания берут одну часть цемента на 6 или более частей песка. Состав подходит для уличных и черновых работ внутри помещений. Так как он не имеет дополнительных добавок, его пластичность и адгезия не позволяет использовать раствор для мелких штукатурных и плиточных работ.

В качестве пластификаторов, улучшающих клейкость и эластичность материала, используется жидкое мыло и клей ПВА. Сделать состав более пластичным помогает глина или карьерный песок. Для осветления и улучшения адгезии строители добавляют в смесь гашеную известь, получая цементно-известковый раствор.

Песок – основной компонент в любом цементном растворе. Его доля в составе в несколько раз превышает содержание бетона, воды и добавок-пластификаторов.

Добавки для цветного шва

Цветовые добавки в цементную смесь служат чисто декоративным целям. Они чаще всего используются при укладке кирпича, чтобы сделать швы более выразительными и эстетичными.

Для придания темного цвета раньше использовали сажу, однако опыт показал, что этот компонент ухудшает технические характеристики цемента. Сегодня в этих целях применяют соли и окиси металлов. Цветовые добавки можно приобрести в строительных магазинах.

Вопрос:

Как придать швам белый цвет?

Ответ:

Белый цвет швов легче всего получить путем увеличения количества извести в растворе. Однако можно приобрести специальную колеровочную смесь с окисью титана или других металлов.

Швы кладки принято делать контрастными по отношению к оттенку кирпича. Для построек из красного кирпича используется белая затирка, для светлого материала – серая или черная.

Красители для цемента выпускают в форме порошка или жидкой краски. Их вносят в состав смеси в процессе ее приготовления и тщательно смешивают с другими ингредиентами.

Пропорции для смешивания

Соотношение элементов в цементном растворе зависит от области применения смеси и марки бетона. К примеру, если нужно получить цементный раствор М100 из бетона М400, соотношение цемента к песку будет 1:4. Таким образом легко высчитать, какое соотношение нужно для получения нужной марки из любого бетона.

Как сделать цементный раствор для разных видов работ (цемент : песок):

  1. Штукатурка: 1:5 с добавлением одной части извести;
  2. Стяжка пола: 1:4 для марки М500, 1:3 для марки М400;
  3. Кирпичная кладка: 1:5 – М3, 1:4 – М5, 1:3 – М8. При этом марка готового раствора должна соответствовать марке кирпича.

Объем воды для приготовления жидкого раствора не должен превышать 60% от общего объема. Обычно она составляет 1-2 части в пропорции.

В процессе смешивания цементный раствор иногда получается слишком жидким. Отложите часть сухой смеси, чтобы иметь возможность добавить ее в случае необходимости.

Время высыхания

Время застывания цементного раствора напрямую зависит от температуры окружающей среды и толщины покрытия. Чем толще слой стяжки и ниже столбик термометра, тем дольше будет сохнуть смесь.

Рассмотрим время застывания при благоприятных условиях:

  • 2–4 часа – схватывание раствора;
  • 12–24 часа – неполное затвердевание цемента, по нему можно ходить, но выполнять отделочные работы не рекомендуется;
  • 10–14 дней – полная просушка покрытия, после которого можно приступать к отделке поверхности плиткой или другим материалом.

Условия застывания легче всего соблюсти внутри помещения – при стяжке пола, штукатурке и других отделочных работах. В квартире отсутствуют перепады температуры, поэтому раствор застывает равномерно и быстро.

Как приготовить цемент при низкой температуре

Идеальная температура для заливки цементного раствора – +200С и выше. Но такие условия не всегда удается создать при строительстве и внешней отделке здания. При минусовой температуре, бетонный раствор ложится комками, а вода в составе превращается в лед. Это напрямую влияет на качество покрытия.

В строительстве существует несколько приемов, которые помогают избежать порчи цементной смеси при минусовых температурах:

  • Специальные противоморозные добавки в состав;
  • Подогрев воды и песка при смешивании бетона;
  • Электрический нагрев покрытия;
  • Укрытие заливки тентом и утепление опалубки.

Для кладки при минусовой температуре (до -7°C) нужно использовать цементный раствор (в соотношении цемента к песку – 1:4) или цементно-кальциевый (1:1:6 – цемент, кальций, песок). Раствор должен быть густо-пластичным, то есть содержать небольшое количество воды.

Сделать электроподогрев бетона своими руками очень сложно. Для этого к поверхности подводят нагревающиеся электрокабели, которые передают тепло бетонному покрытию.

Легче всего утеплить поверхность цемента на открытом воздухе при помощи натянутого тента. Эта конструкция будет удерживать тепло при небольших заморозках.

Использование бетона в отделке дома

Во внутренней отделке квартир и домов бетон используется в трех видах работ:

  • штукатурка стен;
  • заливка стяжки пола;
  • монтаж керамической плитки в санузле.

Раствор для домашней отделки должен быть достаточно пластичным и иметь хорошую адгезию к черновому покрытию. Поэтому в него обязательно добавляют пластификаторы и клейкие добавки.

Для укладки плитки в санузле часто используют готовый клей, однако цементный раствор также хорошо подходит для этих целей. Важно позаботиться о гидроизоляции пола и наличии противогрибковых составов в готовой смеси.

Полезные советы

  1. Подбирайте глубокую ёмкость для раствора, чтобы при размешивании он не выплескивался за стенки;
  2. Вместо раствора для обработки швов кирпича или плитки можно использовать специальные затирки, в таком случае, красители для цемента не потребуются;
  3. Через сутки после стяжки пола, закройте его пленкой, чтобы покрытие не повредилось в процессе ремонта.

Для приготовления смеси рекомендуется использовать речной или карьерный песок. За счет содержания глины в его составе, раствор получится более пластичным и клейким.

Видеоролики о самостоятельном приготовлении цементного раствора

Как замешать бетон своими руками:

Приготовление цветного цементного раствора:

Приготовление цементного раствора не требует специальных навыков и наличия большого количества инструментов. Смесь для стяжки и штукатурки легко сделать своими руками, соблюдая пропорции и используя недорогие пластификаторы. Таким образом можно значительно сэкономить на готовой смеси и получить универсальный состав для разных отделочных работ.

Facebook

Twitter

ВКонтакте

Глава 12.1: Подготовка растворов — Chemistry LibreTexts

  1. Последнее обновление
  2. Сохранить как PDF
  1. Приготовление растворов
    1. Пример 12.1.1
    2. Пример 12.1.2
    3. Пример 12.1.3
  2. Концентрации ионов в растворе
    1. Пример 12.1.4
    2. Ключевые уравнения
    3. Резюме
    4. Ключевые выводы
    5. Концептуальные проблемы
    6. Ответ
    7. Числовые задачи
    8. Ответы
    9. Участники

Цель обучения

  • Для количественного описания концентраций растворов.

В разделе 9.3 мы описали различные способы определения концентрации раствора, молярности (M), моляльности (m), процентных концентраций и мольной доли (X).Количество растворенного вещества, растворенного в определенном количестве растворителя или раствора. раствора описывает количество растворенного вещества, которое содержится в определенном количестве растворителя или раствора. Знание концентрации растворенных веществ важно для контроля стехиометрии реагентов для реакций, протекающих в растворе. В этом разделе описывается, как можно приготовить растворы из основного раствора известной концентрации

.

Приготовление растворов

Чтобы приготовить раствор, который содержит определенную концентрацию вещества, необходимо растворить желаемое количество молей растворенного вещества в достаточном количестве растворителя, чтобы получить желаемый конечный объем раствора.

\ (Молярность раствора = dfrac {моль \: of \: solute} {Объем раствора} \ tag {12.1.1} \)

Рисунок 12.1.1 иллюстрирует эту процедуру для раствора дигидрата хлорида кобальта (II) в этаноле. Обратите внимание, что объем растворителя не указан. Поскольку растворенное вещество занимает пространство в растворе, необходимый объем растворителя почти всегда на меньше , чем желаемый объем раствора. Например, если желаемый объем был 1,00 л, было бы неправильно добавлять 1.00 л воды на 342 г сахарозы, потому что это даст более 1,00 л раствора. Как показано на рисунке 12.1.2, для некоторых веществ этот эффект может быть значительным, особенно для концентрированных растворов.

Рисунок 12.1.1 Приготовление раствора известной концентрации с использованием твердого вещества

Рисунок 12.1.2 Приготовление 250 мл раствора (NH 4 ) 2 Cr 2 O 7 в воде

Растворенное вещество занимает пространство в растворе, поэтому для приготовления 250 мл раствора требуется менее 250 мл воды.

Пример 12.1.1

Раствор на рисунке 12.1.1 содержит 10,0 г дигидрата хлорида кобальта (II), CoCl 2 · 2H 2 O, в этаноле, достаточном для приготовления ровно 500 мл раствора. Какова молярная концентрация CoCl 2 · 2H 2 O?

Дано: масса растворенного вещества и объем раствора

Запрошено: концентрация (M)

Стратегия:

Чтобы найти количество молей CoCl 2 · 2H 2 O, разделите массу соединения на его молярную массу.Рассчитайте молярность раствора, разделив количество молей растворенного вещества на объем раствора в литрах.

Решение:

Молярная масса CoCl 2 · 2H 2 O составляет 165,87 г / моль. Следовательно,

\ (молей \: CoCl_2 \ cdot 2H_2O = \ left (\ dfrac {10.0 \: \ cancel {g}} {165 .87 \: \ cancel {g} / mol} \ right) = 0 .0603 \: mol \)

Объем раствора в литрах

\ (volume = 500 \: \ cancel {mL} \ left (\ dfrac {1 \: L} {1000 \: \ cancel {mL}} \ right) = 0.500 \: L \)

Молярность — это количество молей растворенного вещества на литр раствора, поэтому молярность раствора составляет

.

\ (молярность = \ dfrac {0,0603 \: mol} {0,500 \: L} = 0,121 \: M = CoCl_2 \ cdot H_2O \)

Упражнение

Раствор, показанный на рисунке 12.1.2, содержит 90,0 г (NH 4 ) 2 Cr 2 O 7 в достаточном количестве воды, чтобы получить конечный объем ровно 250 мл. Какова молярная концентрация дихромата аммония?

Ответ: (NH 4 ) 2 Cr 2 O 7 = 1.43 млн

Чтобы приготовить конкретный объем раствора, который содержит указанную концентрацию растворенного вещества, нам сначала нужно вычислить количество молей растворенного вещества в желаемом объеме раствора, используя соотношение, показанное в уравнении 12.1.1. Затем мы переводим количество молей растворенного вещества в соответствующую массу необходимого растворенного вещества. Эта процедура проиллюстрирована в Примере 12.1.2.

Пример 12.1.2

Так называемый раствор D5W, используемый для внутривенного замещения биологических жидкостей, содержит 0.310 М. глюкозы. (D5W представляет собой примерно 5% раствор декстрозы [медицинское название глюкозы] в воде.) Рассчитайте массу глюкозы, необходимую для приготовления пакета D5W объемом 500 мл. Глюкоза имеет молярную массу 180,16 г / моль.

Дано: молярность, объем и молярная масса растворенного вещества

Запрошено: Масса растворенного вещества

Стратегия:

A Рассчитайте количество молей глюкозы в указанном объеме раствора, умножив объем раствора на его молярность.

B Получите необходимую массу глюкозы, умножив количество молей соединения на его молярную массу.

Решение:

A Сначала мы должны рассчитать количество молей глюкозы, содержащихся в 500 мл 0,310 М раствора:

\ (V_L M_ {моль / л} = моль \)

\ (500 \: \ cancel {mL} \ left (\ dfrac {1 \: \ cancel {L}} {1000 \: \ cancel {mL}} \ right) \ left (\ dfrac {0 .310 \: моль \: глюкоза} {1 \: \ cancel {L}} \ right) = 0.155 \: моль \: глюкоза \)

B Затем мы переводим количество молей глюкозы в требуемую массу глюкозы:

\ (масса \: of \: глюкоза = 0,155 \: \ cancel {моль \: глюкоза} \ left (\ dfrac {180.16 \: g \: глюкоза} {1 \: \ cancel {моль \: глюкоза}} \ справа) = 27,9 \: г \: глюкоза \)

Упражнение

Другой раствор, обычно используемый для внутривенных инъекций, — это физиологический раствор, 0,16 М раствор хлорида натрия в воде. Рассчитайте массу хлорида натрия, необходимую для приготовления 250 мл физиологического раствора.

Ответ: 2,3 г NaCl

Раствор желаемой концентрации также можно приготовить путем разбавления небольшого объема более концентрированного раствора дополнительным растворителем. Базовый раствор — это коммерчески приготовленный раствор известной концентрации, который часто используется для этой цели. Разбавление исходного раствора является предпочтительным, поскольку альтернативный метод взвешивания крошечных количеств растворенного вещества трудно осуществить с высокой степенью точности. Разбавление также используется для приготовления растворов из веществ, которые продаются в виде концентрированных водных растворов, таких как сильные кислоты.

Процедура приготовления раствора известной концентрации из основного раствора показана на рисунке 12.1.3. Это требует расчета желаемого количества молей растворенного вещества в конечном объеме более разбавленного раствора, а затем расчета объема исходного раствора, который содержит это количество растворенного вещества. Помните, что разбавление данного количества основного раствора растворителем не приводит к изменению числа , а не количества молей присутствующего растворенного вещества. Следовательно, соотношение между объемом и концентрацией основного раствора и объемом и концентрацией желаемого разбавленного раствора составляет

\ ((V_s) (M_s) = моли \: of \: solute = (V_d) (M_d) \ tag {12.1.2} \)

, где нижние индексы s и d обозначают исходный и разбавленный растворы соответственно. Пример 5 демонстрирует расчеты, связанные с разбавлением концентрированного исходного раствора.

Рисунок 12.1.3 Приготовление раствора известной концентрации путем разбавления исходного раствора (a) Объем ( V s ), содержащий желаемые моли растворенного вещества (M s ), измеряют из исходного раствора. раствор известной концентрации.(b) Отмеренный объем исходного раствора переносят во вторую мерную колбу. (c) Измеренный объем во второй колбе затем разбавляется растворителем до объемной отметки [( V s ) (M s ) = ( V d ) (M d ). ].

Пример 12.1.3

Какой объем 3,00 М исходного раствора глюкозы необходим для приготовления 2500 мл раствора D5W в Примере 4?

Дано: Объем и молярность разбавленного раствора

Запрошено: объем основного раствора

Стратегия:

A Рассчитайте количество молей глюкозы в указанном объеме разбавленного раствора, умножив объем раствора на его молярность.

B Чтобы определить необходимый объем исходного раствора, разделите количество молей глюкозы на молярность исходного раствора.

Решение:

A Раствор D5W в Примере 4 содержал 0,310 М глюкозы. Начнем с использования уравнения 12.1.2 для расчета количества молей глюкозы, содержащихся в 2500 мл раствора:

\ (моль \: глюкоза = 2500 \: \ cancel {mL} \ left (\ dfrac {1 \: \ cancel {L}} {1000 \: \ cancel {mL}} \ right) \ left (\ dfrac { 0.310 \: моль \: глюкоза} {1 \: \ cancel {L}} \ right) = 0,775 \: моль \: глюкоза \)

B Теперь мы должны определить объем исходного раствора 3,00 M, который содержит это количество глюкозы:

\ (объем \: of \: stock \: soln = 0,775 \: \ cancel {mol \: gluosis} \ left (\ dfrac {1 \: L} {3 .00 \: \ cancel {mol \: глюкоза}} \ right) = 0,258 \: L \: или \: 258 \: mL \)

При определении необходимого объема исходного раствора мы должны были разделить желаемое количество молей глюкозы на концентрацию исходного раствора, чтобы получить соответствующие единицы.Кроме того, количество молей растворенного вещества в 258 мл исходного раствора такое же, как количество молей в 2500 мл более разбавленного раствора; изменилось только количество растворителя . Полученный ответ имеет смысл: разбавление основного раствора примерно в 10 раз увеличивает его объем примерно в 10 раз (258 мл → 2500 мл). Следовательно, концентрация растворенного вещества должна уменьшиться примерно в 10 раз, как это происходит (3,00 M → 0,310 M).

Мы также могли решить эту проблему за один шаг, решив уравнение 12.1,2 для В с и подставив соответствующие значения:

\ (V_s = \ dfrac {(V_d) (M_d)} {M_s} = \ dfrac {(2 .500 \: L) (0,310 \: \ cancel {M})} {3 .00 \: \ отменить {M}} = 0,258 \: L \)

Как мы уже отмечали, часто существует несколько правильных способов решения проблемы.

Упражнение

Какой объем 5,0 М маточного раствора NaCl необходим для приготовления 500 мл физиологического раствора (0,16 М NaCl)?

Ответ: 16 мл

Концентрации ионов в растворе

В разделе 9. {2-} (водный) \ tag {12.1.2} \)

Таким образом, 1 моль единиц формулы дихромата аммония растворяется в воде с образованием 1 моль анионов Cr 2 O 7 2- и 2 моль катионов NH 4 + (см. Рисунок 12.1.4).

Рисунок 12.1.4 Растворение 1 моля ионного соединения В этом случае растворение 1 моля (NH 4 ) 2 Cr 2 O 7 дает раствор, содержащий 1 моль Cr 2 O 7 2- ионов и 2 моль NH 4 + ионов.(Молекулы воды не показаны с молекулярной точки зрения для ясности.)

Когда мы проводим химическую реакцию с использованием раствора соли, например дихромата аммония, нам необходимо знать концентрацию каждого иона, присутствующего в растворе. Если раствор содержит 1,43 M (NH 4 ) 2 Cr 2 O 7 , то концентрация Cr 2 O 7 2- также должна быть 1,43 M, потому что существует один Cr 2 O 7 2- ионов на формульную единицу.Однако на формульную единицу приходится два иона NH 4 + , поэтому концентрация ионов NH 4 + составляет 2 × 1,43 M = 2,86 М. Поскольку каждая формульная единица (NH 4 ) 2 Cr 2 O 7 при растворении в воде образует три иона (2NH 4 + + 1Cr 2 O 7 2-), общая концентрация ионов в решение 3 × 1,43 M = 4,29 M.

Пример 12.1,4

Каковы концентрации всех веществ, полученных из растворенных веществ, в этих водных растворах?

  1. 0,21 М NaOH
  2. 3,7 M (CH 3 ) CHOH
  3. 0,032 M дюйм (NO 3 ) 3

Дано: молярность

Запрошено: концентрации

Стратегия:

A Классифицируйте каждое соединение как сильнодействующий электролит или как неэлектролит. — (водн.) \)

B Поскольку каждая формульная единица NaOH производит один ион Na + и один ион OH , концентрация каждого иона такая же, как концентрация NaOH: [Na + ] = 0.21 M и [OH ] = 0,21 M.

  • A Формула (CH 3 ) 2 CHOH представляет собой 2-пропанол (изопропиловый спирт) и содержит группу –OH, поэтому это спирт. Напомним из раздела 9.1, что спирты — это ковалентные соединения, которые растворяются в воде с образованием растворов нейтральных молекул. Таким образом, спирты не являются электролитами.

    B Таким образом, единственными растворенными веществами в растворе являются (CH 3 ) 2 молекул CHOH, поэтому [(CH 3 ) 2 CHOH] = 3.- (водн.) \)

    B Одна формульная единица In (NO 3 ) 3 дает один ион In 3 + и три иона NO 3 , поэтому 0,032 M In (NO 3 ) 3 раствор содержит 0,032 M In 3 + и 3 × 0,032 M = 0,096 M NO 3 , то есть [In 3 + ] = 0,032 M и [NO 3 ] = 0,096 М.

  • Упражнение

    Каковы концентрации всех веществ, полученных из растворенных веществ, в этих водных растворах?

    1. 0.0012 M Ba (OH) 2
    2. 0,17 M Na 2 SO 4
    3. 0,50 M (CH 3 ) 2 CO, широко известный как ацетон

    Ответ:

    1. [Ba 2 + ] = 0,0012 М; [OH ] = 0,0024 M
    2. [Na + ] = 0,34 М; [SO 4 2−] = 0,17 M
    3. [(CH 3 ) 2 CO] = 0,50 M

    Ключевые уравнения

    соотношение между объемом и концентрацией основного и разбавленного растворов

    Уравнение 12.1.2: \ ((V_s) (M_s) = моль \: of \: solute = (V_d) (M_d) \)

    Сводка

    Концентрация вещества — это количество растворенного вещества, присутствующего в данном количестве раствора. Концентрации обычно выражаются как молярность , количество молей растворенного вещества в 1 л раствора. Растворы известной концентрации могут быть приготовлены либо путем растворения известной массы растворенного вещества в растворителе и разбавления до желаемого конечного объема, либо путем разбавления соответствующего объема более концентрированного раствора (исходный раствор ) до желаемого конечного объема.

    Ключевые вынос

    • Концентрации раствора обычно выражаются в виде молярности и могут быть получены путем растворения известной массы растворенного вещества в растворителе или разбавления исходного раствора.

    Концептуальные проблемы

    1. Какое из представлений лучше всего соответствует 1 М водному раствору каждого соединения? Обоснуйте свои ответы.

      1. NH 3
      2. HF
      3. CH 3 CH 2 CH 2 OH
      4. Na 2 SO 4

    2. Какое из представлений, показанных в задаче 1, лучше всего соответствует 1 М водному раствору каждого соединения? Обоснуйте свои ответы.

      1. CH 3 CO 2 H
      2. NaCl
      3. Na 2 S
      4. Na 3 PO 4
      5. ацетальдегид
    3. Можно ли ожидать, что 1,0 М раствор CaCl 2 будет лучше проводить электричество, чем 1,0 М раствор NaCl? Почему или почему нет?

    4. Альтернативный способ определения концентрации раствора — моляльность , сокращенно м .Моляльность определяется как количество молей растворенного вещества в 1 кг растворителя . Чем это отличается от молярности? Ожидаете ли вы, что 1 M раствор сахарозы будет более или менее концентрированным, чем 1 m раствор сахарозы? Поясните свой ответ.

    5. Каковы преимущества использования решений для количественных расчетов?

    Ответ

    1. Если количество вещества, необходимое для реакции, слишком мало для точного взвешивания, использование раствора вещества, в котором растворенное вещество диспергировано в гораздо большей массе растворителя, позволяет химикам измерить количество вещества. вещество, точнее.

    Числовые задачи

    1. Рассчитайте количество граммов растворенного вещества в 1.000 л каждого раствора.

      1. 0,2593 M NaBrO 3
      2. 1,592 М ННО 3
      3. 1,559 М уксусная кислота
      4. 0,943 M йодат калия
    2. Рассчитайте количество граммов растворенного вещества в 1.000 л каждого раствора.

      1. 0.1065 Мбайн 2
      2. 1,135 M Na 2 SO 4
      3. 1,428 млн NH 4 Br
      4. 0,889 М ацетат натрия
    3. Если все растворы содержат одно и то же растворенное вещество, какой раствор содержит большую массу растворенного вещества?

      1. 1,40 л 0,334 М раствора или 1,10 л 0,420 М раствора
      2. 25,0 мл 0,134 М раствора или 10,0 мл 0,295 М раствора
      3. 250 мл 0.489 М раствор или 150 мл 0,769 М раствора
    4. Заполните следующую таблицу для 500 мл раствора.

      Соединение Масса (г) Моль Концентрация (M)
      сульфат кальция 4,86 ​​
      уксусная кислота 3.62
      дигидрат иодистого водорода 1,273
      бромид бария 3,92
      глюкоза 0,983
      ацетат натрия 2.42
    5. Какая концентрация каждого вида присутствует в следующих водных растворах?

      1. 0,489 моль NiSO 4 в 600 мл раствора
      2. 1,045 моль бромида магния в 500 мл раствора
      3. 0,146 моль глюкозы в 800 мл раствора
      4. 0,479 моль CeCl 3 в 700 мл раствора
    6. Какая концентрация каждого вида присутствует в следующих водных растворах?

      1. 0.324 моль K 2 MoO 4 в 250 мл раствора
      2. 0,528 моль формиата калия в 300 мл раствора
      3. 0,477 моль KClO 3 в 900 мл раствора
      4. 0,378 моль йодида калия в 750 мл раствора
    7. Какова молярная концентрация каждого раствора?

      1. 8,7 г бромида кальция в 250 мл раствора
      2. 9,8 г сульфата лития в 300 мл раствора
      3. 12.4 г сахарозы (C 12 H 22 O 11 ) в 750 мл раствора
      4. 14,2 г гексагидрата нитрата железа (III) в 300 мл раствора
    8. Какова молярная концентрация каждого раствора?

      1. 12,8 г гидросульфата натрия в 400 мл раствора
      2. 7,5 г гидрофосфата калия в 250 мл раствора
      3. 11,4 г хлорида бария в 350 мл раствора
      4. 4.3 г винной кислоты (C 4 H 6 O 6 ) в 250 мл раствора
    9. Приведите концентрацию каждого реагента в следующих уравнениях, принимая 20,0 г каждого реагента и объем раствора 250 мл для каждого реагента.

      1. BaCl 2 (водн.) + Na 2 SO 4 (водн.) →
      2. Ca (OH) 2 (водн.) + H 3 PO 4 (водн.) →
      3. Al (NO 3 ) 3 (водн.) + H 2 SO 4 (водн.) →
      4. Pb (NO 3 ) 2 (водн.) + CuSO 4 (водн.) →
      5. Al (CH 3 CO 2 ) 3 (водн.) + NaOH (водн.) →
    10. На эксперимент потребовалось 200.0 мл 0,330 М раствора Na 2 CrO 4 . Для приготовления этого раствора использовали исходный раствор Na 2 CrO 4 , содержащий 20,0% растворенного вещества по массе с плотностью 1,19 г / см 3 . Опишите, как приготовить 200,0 мл 0,330 М раствора Na 2 CrO 4 с использованием исходного раствора.

    11. Гипохлорит кальция [Ca (OCl) 2 ] — эффективное дезинфицирующее средство для одежды и постельного белья.Если в растворе концентрация Ca (OCl) 2 составляет 3,4 г на 100 мл раствора, какова молярность гипохлорита?

    12. Фенол (C 6 H 5 OH) часто используется в качестве антисептика в жидкостях для полоскания рта и пастилках для горла. Если в жидкости для полоскания рта концентрация фенола составляет 1,5 г на 100 мл раствора, какова молярность фенола?

    13. Если таблетка, содержащая 100 мг кофеина (C 8 H 10 N 4 O 2 ), растворяется в воде с получением 10.0 унций раствора, какова молярная концентрация кофеина в растворе?

    14. На этикетке с определенным лекарством есть инструкция по добавлению 10,0 мл стерильной воды, в которой указано, что каждый миллилитр полученного раствора будет содержать 0,500 г лекарства. Если пациенту назначена доза 900,0 мг, сколько миллилитров раствора следует ввести?

    ответы

    1. 1.74 × 10 −3 M кофеин

    Авторы

    Изменено Джошуа Халперн, Скотт Синекс и Скотт Джонсон

    Химический, молярный и массовый процент

    Растворы представляют собой однородные (равномерно распределенные) смеси двух или более химических веществ. Растворы могут существовать в виде твердых тел, жидкостей или газов.

    Все растворы содержат растворитель и одно или несколько растворенных веществ. Растворитель, часто вода, является наиболее распространенным химическим веществом.Растворенное вещество — это менее распространенные химические вещества.

    Создание решений

    Как растворить твердые частицы и уменьшить масштабы экспериментов

    Для экспериментов вам часто нужно растворять растворенные вещества в твердой форме, чтобы получить растворы определенной силы (сила измеряется по диссоциации ионов). Запланируйте один час на каждые 2-4 раствора, которые вам нужно приготовить. Вам понадобятся весы для взвешивания растворенного вещества и градуированный цилиндр для измерения растворителя (если это вода).

    Во-первых, определите концентрацию (массовые проценты или молярность, см. Ниже) и количество (миллилитры) раствора, которое вам нужно в лабораторной процедуре. Во-вторых, рассчитайте необходимое количество растворенного вещества в граммах, используя одну из формул, приведенных ниже. Затем взвесьте растворенное вещество и добавьте его в стакан для смешивания. Наконец, измерьте необходимый объем воды в миллилитрах с помощью градуированного цилиндра и добавьте его в стакан. Размешайте раствор, пока все химическое вещество не растворится.

    Разбейте твердые куски химического вещества ступкой и пестиком или осторожно раздавив молотком в полиэтиленовом пакете.Химические вещества растворяются быстрее при осторожном нагревании раствора и перемешивании.

    Вы можете рассмотреть возможность уменьшения масштаба, когда эксперименты требуют больших количеств химикатов. Уменьшение масштаба снижает риски безопасности, химические затраты и удаление отходов.

    Большинство экспериментов можно уменьшить, разделив растворенное вещество и растворитель на коэффициент по вашему выбору. Например, эксперимент, требующий 50 г растворителя и 250 мл воды, можно уменьшить в 10 раз, чтобы использовать только 5 г растворителя и 25 мл воды.Вы можете упростить масштабирование, используя мензурки, пробирки и другое измерительное оборудование меньшего размера.

    При приготовлении химических растворов всегда используйте соответствующее защитное оборудование.

    Как приготовить молярные растворы

    Молярные (М) растворы основаны на количестве молей химического вещества в одном литре раствора. Моль состоит из 6,02 × 10 23 молекул или атомов. Молекулярный вес (MW) — это вес одного моля химического вещества. Определите молекулярную массу с помощью таблицы Менделеева, добавив атомную массу каждого атома в химическую формулу.

    Пример: для молекулярной массы CaCl 2 добавьте атомную массу Ca (40,01) к атомной массе двух Cl (2 x 35,45), чтобы получить 110,91 г / моль. Следовательно, 1М раствор CaCl 2 состоит из 110,91 г CaCl 2 , растворенных в воде, достаточной для приготовления одного литра раствора.

    После того, как молекулярная масса растворенного вещества известна, масса химического вещества, которое должно раствориться в растворе для молярного раствора менее 1M, рассчитывается по формуле:

    • грамма химического вещества = (молярность раствора в моль / литр) x (молекулярная масса химического вещества в г / моль) x (мл раствора) ÷ 1000 мл / литр

    Например, чтобы сделать 100 мл 0.1 M раствор CaCl 2 , используйте предыдущую формулу, чтобы узнать, сколько CaCl 2 вам нужно:

    • грамма CaCl 2 = (0,1) x (110,91) x (100) ÷ (1000) = 1,11 г

    Теперь вы можете приготовить раствор: растворите 1,11 г CaCl 2 в воде, достаточной для получения 100 мл раствора. Необходимое количество воды будет чуть меньше 100 мл.

    Для приготовления молярных растворов используются весы и мерная колба. Процедура приготовления молярного раствора с использованием мерной колбы на 100 мл следующая:

    1. Рассчитайте вес растворенного вещества, необходимый для приготовления 100 мл раствора, используя приведенную выше формулу.
    2. Взвесьте необходимое количество растворенного вещества на весах.
    3. Перенесите растворенное вещество в чистую сухую мерную колбу на 100 мл.
    4. Медленно добавьте дистиллированную воду в мерную колбу. При этом промойте все растворенное вещество на дне колбы. Продолжайте добавлять воду, пока не дойдете до отметки 100 мл на горлышке колбы.
    5. Поместите пробку в колбу и осторожно покрутите колбу, пока все растворенное вещество не растворится.

    Если у вас нет мерной колбы, вы можете использовать мерный цилиндр на 100 мл.Просто добавьте растворенное вещество в мерный цилиндр, а затем добавьте дистиллированную воду, пока не достигнете отметки 100 мл на боковой стороне цилиндра.

    Как приготовить растворы в процентах по массе

    В растворах массовых процентов вес растворенного вещества делится на вес раствора (растворенное вещество + вода) и умножается на 100. Поскольку плотность воды составляет 1 г / мл, формула для расчета количества растворенного вещества, которое должно быть смешанным для получения массового процента раствора составляет:

    • грамма растворенного вещества = (вес.% Раствора) x (мл воды) ÷ (100 — вес.% Раствора)

    В качестве примера, чтобы приготовить 100 мл 10% раствора NaCl (поваренной соли), используйте предыдущую формулу, чтобы узнать, сколько NaCl вам нужно:

    • грамма NaCl = (10) x (100) ÷ (100-10) = 11.1 г

    Теперь вы можете приготовить свой раствор: растворите 11,1 г NaCl в 100 мл воды.

    Создайте собственное решение для этих проектов:

    Замороженные пузыри

    Радужная реакция

    Биологический тест на токсичность

    Гальваника: ключ с медным покрытием

    Приготовление растворов

    Приготовление на водной основе
    Решения данной молярности

    Молярность — полезный способ описания концентраций растворенных веществ в
    решение, а также источник полезных коэффициентов пересчета для конвертации
    между объемом (легко измеримое свойство растворов) и молями растворенного вещества.

    Теперь посмотрим, как решения
    определенной молярности.

    Представьте, что вы готовите
    провести эксперимент, требующий двух растворов, 0,500 M Na 2 CO 3
    и 1,50 М HCl. Типичный склад химикатов, вероятно, не будет хранить
    решения этих типов, поэтому вам нужно будет составлять их из запасов
    это доступно. На складе, вероятно, есть чистый карбонат натрия, поэтому
    0.500 M Na 2 CO 3 раствор может быть получен из чистого Na 2 CO 3
    и вода. На складе почти наверняка не будет чистого хлористого водорода,
    однако, потому что это газ, который трудно и опасно хранить. Вместо,
    вы, вероятно, найдете концентрированный раствор соляной кислоты, который составляет 12
    M HCl. Этот раствор можно разбавить до 1,50 М HCl.

    Приведенное выше обсуждение предполагает
    что есть два основных способа приготовления растворов с заданной молярностью:

    • Растворы можно приготовить, исходя из чистого растворенного вещества и воды.
    • Растворы можно приготовить из более концентрированных растворов.

    Изготовление растворов из чистых твердых частиц

    Следующие шаги описывают
    процедура приготовления раствора определенной молярности из чистого твердого
    вещество.

    • Сначала взвесьте правильную массу растворенного вещества.
    • Растворите растворенное вещество в воде, сохраняя объем меньше, чем
      желаемый общий объем раствора.
    • Разбавьте раствор до желаемого общего объема раствора.

    Нажмите
    здесь, чтобы увидеть Пример 1.

    Нажмите
    здесь, чтобы увидеть образец листа исследования проблем
    которые просят вас описать, как приготовить водный раствор, начиная с чистого
    твердый.

    Нажмите
    здесь, чтобы увидеть Упражнение 1.

    Изготовление
    Растворы из более концентрированных растворов

    Потому что это непрактично для
    типичная лаборатория для приготовления растворов соляной кислоты из чистого HCl, химик
    чаще заказывают раствор соляной кислоты, а не чистый водород
    хлорид от химической компании-поставщика.Растворы соляной кислоты a
    химик может использовать в реакциях концентрации 1 M HCl или 6 M HCl,
    но растворы такой концентрации занимали бы неудобное количество
    место для хранения на складе химии. Следовательно, концентрированная соляная
    раствор кислоты, обычно около 12 M HCl, покупается и хранится, и разбавляется
    воды, когда требуется более разбавленный раствор. Концентрации типичных
    концентрированные растворы кислот и щелочной аммиак приведены в таблице.
    ниже.

    Типичные концентрации концентрированных кислот и аммиака

    Вещество

    Молярность

    Массовый процент

    Соляная кислота, HCl (водн.)

    12,1 млн

    38.7%

    Серная кислота, H 2 SO 4

    от 17,8 млн до 18,4 млн

    95% -98%

    Азотная кислота, HNO 3

    15,8 млн

    69,71%

    Уксусная кислота, HC 2 H 3 O 2

    17.4 м

    99,7%

    Аммиак, NH 3 (водн.)

    14,8 млн

    28%

    Ниже приводится
    общая процедура разбавления относительно концентрированного раствора для образования
    тот, который более разбавлен. (Для
    разбавление почти чистых кислот, таких как концентрированная серная кислота,
    что составляет 95-98% H 2 SO 4 .Мы ограничимся нашими примерами
    к разбавлению растворов, которые уже содержат значительное количество
    вода.)

    • Добавьте правильный объем более концентрированного раствора в
      прибор для измерения объема, например мерная колба. (См. Пример
      15.7 для примера того, как рассчитывается этот объем.)
    • Добавьте воды, пока объем не достигнет желаемого значения.

    Нажмите
    здесь, чтобы увидеть Пример 2.

    Как упоминалось в
    Пример №2, ярлык может использоваться для решения проблем, в которых
    относительно концентрированный раствор необходимо разбавить для получения раствора
    это менее концентрировано. Когда более концентрированный раствор разбавляется на
    при смешивании с водой количество растворенного вещества не меняется. Таким образом, число
    молей растворенного вещества в концентрированном растворе равно количеству
    молей растворенного вещества в разбавленном растворе.

    # моль растворенного в
    концентрированный раствор = # моль растворенного вещества в разбавленном растворе

    Молей растворенного вещества могут быть
    рассчитывается путем умножения объема раствора на молярность
    решение.

    или

    или
    V конц сол M конц
    soln
    = V dil soln M dil
    солнечный

    Это знание приводит к
    легко запоминающееся уравнение, которое мы можем использовать для решения задач этого
    тип.Мы будем называть эти проблемы проблемами разбавления.

    M C V C
    = M D V D

    M C =
    молярность более концентрированного раствора

    V C = объем
    более концентрированного раствора

    M D =
    молярность более разбавленного раствора

    V D = объем
    более разбавленного раствора

    Если вы
    знать это уравнение и распознавать ситуации, к которым оно применимо,
    это может сэкономить вам довольно много времени.Например, его можно использовать вместо
    размерного анализа в работу Пример №3.

    Нажмите
    здесь, чтобы увидеть альтернативное решение примера 2.

    а
    образец
    учебный лист по проблемам разбавления.

    Нажмите
    здесь, чтобы увидеть Пример 3.

    Щелкните здесь, чтобы просмотреть Упражнение 2.

    Нажмите
    здесь, чтобы увидеть пример 4.

    Щелкните здесь, чтобы просмотреть общий образец листа исследования для
    проблемы, которые просят вас описать, как решить конкретную
    молярность.

    Нажмите
    здесь, чтобы увидеть Упражнение 3.

    Подготовка растворов

    Подготовка решений

    Растворы обычно готовятся в лаборатории из твердых материалов, жидкостей или других растворов. Приведенные ниже описания предполагают знание расчетов, необходимых для определения концентраций раствора, способность точно взвешивать твердые частицы и пипетировать жидкости.

    Из твердого материала

    (1) Определите концентрацию и количество раствора, необходимые для эксперимента.
    (2) Рассчитайте количество растворенного вещества, необходимое для приготовления желаемого раствора.
    (3) Взвесьте количество растворенного вещества, рассчитанное на этапе (2), и получите мерную колбу соответствующего объема.
    (4) Добавьте растворенное вещество в мерную колбу.
    (5) Заполните мерную колбу примерно на две трети, закройте пробкой и перемешайте. Для этого переверните колбу, встряхните и верните колбу в вертикальное положение.Сделайте это десять раз. Обязательно удерживайте пробку в колбе.
    (6) Осторожно наполните колбу до отметки, нанесенной на горлышко колбы. При необходимости используйте промывочную бутылку или капельницу для лекарств.
    (7) Тщательно перемешайте раствор, плотно закрыв колбу и перевернув ее десять-двенадцать раз.

    Из жидкости или другого раствора

    (1) Определите концентрацию и количество раствора, необходимые для эксперимента.
    (2) Рассчитайте количество исходного раствора или жидкости, необходимое для приготовления желаемого раствора (маточный раствор — это раствор с известной концентрацией, превышающей раствор, который вы готовите).
    (3) Используйте пипетку для измерения количества раствора или жидкости, рассчитанного на шаге (2).
    (4) Добавьте раствор или жидкость в мерную колбу соответствующего объема.
    (5) Заполните мерную колбу примерно на две трети и перемешайте.
    (6) Осторожно наполните колбу до отметки, нанесенной на горлышко колбы. При необходимости используйте промывочную бутылку или капельницу для лекарств.
    (7) Тщательно перемешайте раствор, плотно закрыв колбу и перевернув ее десять-двенадцать раз.


    Приготовление растворов в лаборатории

    Приготовление растворов — важная процедура, включенная практически во все биологические и химические эксперименты, проводимые по всему миру.

    Раствор состоит из вещества, растворенного в жидкости. Растворенное вещество известно как растворенное вещество, а основная жидкость — как растворитель. Полученная однородная смесь называется раствором.

    Растворы можно описать по концентрации растворенных веществ, то есть по количеству растворенных веществ на единицу раствора.

    Принятие решений может быть основным лабораторным навыком, но плохая техника может означать разницу между успешным или неудачным экспериментом.

    Первое, что нужно учитывать при принятии решений — это безопасность. Важно принять соответствующие меры предосторожности, например надеть перчатки и лабораторный халат, в зависимости от типа химикатов, с которыми вы работаете.

    Есть много разных способов найти решение. Это видео продемонстрирует наиболее распространенный способ приготовления раствора на водной основе.

    Сначала определите количество молей растворенного вещества, необходимое для достижения желаемой концентрации в данном объеме раствора.Затем сократите это значение в граммах, используя молекулярную массу или количество граммов на моль химического вещества.

    Химические вещества можно взвесить с помощью цифровых весов и весовой лодки.

    Затем можно использовать градуированный цилиндр для измерения объема очищенной воды, который составляет примерно три четверти окончательного объема раствора.

    Крайне важно, чтобы водные растворы готовились на очищенной воде, а не на водопроводной воде. Невыполнение этого требования может поставить под угрозу качество не только решения, но и потенциально нескольких последующих экспериментов.

    На этом этапе очищенная вода должна быть перенесена в стакан, содержащий стержень для перемешивания на магнитной пластине для перемешивания.

    Затем измеренные растворенные вещества можно добавить в очищенную воду при перемешивании. Перемешивание смеси способствует растворению растворенного вещества. Для этой цели также можно использовать нагревание.

    После того, как все растворенные вещества растворятся в растворителе, можно отрегулировать pH раствора с помощью pH-метра. Чтобы повысить pH, добавьте к перемешиваемому раствору разбавленный гидроксид натрия.Чтобы снизить pH, добавьте разбавленную соляную кислоту. Обязательно медленно добавляйте кислоту или основание, так как pH может быстро измениться.

    Бумагу для измерения pH

    также можно использовать для измерения pH раствора, однако использование калиброванного pH-метра обеспечивает более точное измерение.

    Затем раствор с помощью воронки переливают в мерную колбу, чтобы довести его до конечного объема. Добавление количества, достаточного для достижения этого объема, называется Q.S.

    Убедитесь, что мениск совпадает с отметкой на мерной колбе.В водном растворе мениск вогнутый, и его следует читать в самой нижней точке кривой.

    При проведении биологических исследований, особенно с участием живых клеток, может потребоваться стерилизация растворов перед использованием. Это можно сделать с помощью автоклавирования, при котором раствор подвергается воздействию пара высокой температуры под высоким давлением.

    В качестве альтернативы раствор можно стерилизовать, пропустив фильтр 0,22 микрон, который исключит любые бактериальные клетки.

    Теперь, когда у вас есть фундаментальное понимание того, как находить решения, пора взглянуть на некоторые широко используемые решения в лаборатории и их приложения.

    В биологических исследованиях разработаны многочисленные решения, имитирующие физиологические жидкости. Эти растворы забуферены, что означает, что они сопротивляются изменению pH в определенном диапазоне; обычно pH поддерживается на уровне 7,4 для имитации внутриклеточных и внеклеточных жидкостей.

    Солевой раствор с фосфатным буфером или PBS, обычно используемый в биологических исследованиях буфер, имитирующий физиологический pH и осмолярность. Осмолярность относится к общему количеству молей растворенного вещества в растворе. Например, раствор, содержащий 1 моль NaCl, имеет 2 осмоля растворенного вещества, потому что ионы натрия и хлорида диссоциируют в растворе.PBS имеет концентрации ионов, которые близко соответствуют таковым в клетках, что делает его изотоническим раствором, а это означает, что количество растворенного вещества вне клетки эквивалентно тому, что находится внутри клетки. PBS состоит из нескольких различных солей в воде, включая соли с фосфатными группами, которые поддерживают постоянный pH в диапазоне от 7,2 до 7,6.

    Обычно PBS в лаборатории используется для промывания клеток и разбавления биомолекул, таких как белок.

    Искусственная спинномозговая жидкость, или ACSF, имитирует концентрацию электролитов спинномозговой жидкости.Этот раствор должен быть свежеприготовленным, а pH, осмолярность и ионный состав следует тщательно контролировать, чтобы соответствовать условиям in vivo.

    ACSF обычно используется в электрофизиологических исследованиях для подготовки срезов мозга и их перфузии во время экспериментов. Он также может служить в качестве внеклеточного раствора во время измерения с помощью зажима.

    Раствор Рингера — это изотонический физиологический раствор со сбалансированным pH, используемый в биологических исследованиях. Он обычно используется в экспериментах in vitro с органами и тканями.

    Вы только что посмотрели, как JoVE знакомится с решениями. В этом видео мы рассмотрели, как приготовить раствор от начала до конца… в том числе как определить необходимое количество растворенного вещества (A), как правильно контролировать качество раствора (B) и методы стерилизации (C). Мы также рассмотрели некоторые общие решения, а также их применение в биологических исследованиях (D).

    Спасибо за просмотр и не забывайте всегда использовать правильную технику при принятии решений.

    Требуется подписка.Пожалуйста, порекомендуйте JoVE своему библиотекарю.

    Руководство по приготовлению раствора | Carolina.com

    Carolina предлагает много типов готовых решений, но некоторые учителя предпочитают создавать свои собственные. Если это вас интересует, продолжайте читать. Это краткое руководство предоставит вам информацию, необходимую для создания ряда решений, обычно используемых в учебных лабораториях.

    Давайте рассмотрим некоторые соображения безопасности:

    • Всегда используйте соответствующие средства индивидуальной защиты (СИЗ) при работе с химическими веществами и приготовлении растворов.Используйте это руководство, чтобы выбрать СИЗ, подходящие для ваших нужд.
    • Перед использованием дважды прочтите этикетку на химическом веществе. Прочтите его, когда возьмете химикат с полки, и еще раз перед тем, как удалить какое-либо химическое вещество из бутылки.
    • При использовании концентрированных химикатов для приготовления растворов убедитесь, что вы медленно добавляете более концентрированный раствор к менее концентрированному. Обратный порядок действий может привести к закипанию и разбрызгиванию раствора.

    Приготовление раствора

    Молярные растворы

    Молярность (M) означает количество молей растворенного вещества на литр раствора.Чтобы приготовить 1 М раствор, медленно добавьте 1 формульную массу соединения в чистую мерную колбу объемом 1 л, наполовину заполненную дистиллированной или деионизированной водой. Дайте составу полностью раствориться, при необходимости осторожно покачивая колбу. Как только растворенное вещество полностью растворится и раствор достигнет комнатной температуры, разбавьте водой до метки. Вставьте пробку и, положив большой палец на пробку и положив руку на горлышко колбы,. переверните колбу несколько раз, чтобы перемешать. Вот пара примеров использования этого метода:

    • Чтобы приготовить 1 М раствор гидроксида натрия, медленно добавьте 40 г гидроксида натрия к 500 мл дистиллированной или деионизированной воды в мерной колбе объемом 1 л.Когда твердые вещества полностью растворятся и раствор достигнет комнатной температуры, разбавьте до отметки, вставьте и закрепите пробку большим пальцем и несколько раз переверните колбу для перемешивания.

    • Чтобы приготовить 1 М раствор уксусной кислоты, растворите 60,05 г уксусной кислоты в 500 мл дистиллированной или деионизированной воды в мерной колбе объемом 1 л. Поскольку уксусная кислота является жидкостью, ее можно также измерить по объему. Разделите массу кислоты на ее плотность (1,049 г / мл), чтобы определить объем (57.24 мл). Для приготовления раствора используйте 60,05 г или 57,24 мл уксусной кислоты. Осторожно покрутите колбу, чтобы перемешать раствор. Когда раствор достигнет комнатной температуры, разбавьте его до отметки, вставьте и зафиксируйте пробку большим пальцем и несколько раз переверните колбу для перемешивания.

    Разведения

    При приготовлении разведения определите требуемый объем и молярную концентрацию полученного раствора. Используйте следующее уравнение, чтобы определить, сколько концентрированного реагента необходимо для приготовления разбавленного раствора,

    M реагент × V реагент = M разбавление × V разбавление

    , где M — молярность, а V — объем.

    Медленно добавьте рассчитанный объем концентрированного реагента в мерную колбу подходящего размера, наполовину заполненную дистиллированной или деионизированной водой, и встряхните колбу для перемешивания. Когда раствор достигнет комнатной температуры, разбавьте его водой до метки, вставьте и закройте пробку и несколько раз переверните колбу для перемешивания.

    Например, какой объем 10 М уксусной кислоты требуется для приготовления 1,0 л 0,50 М уксусной кислоты?

    10 M × V реагент = 0.50 M × 1,0 л
    V реагент = 0,050 л = 50 мл

    Для приготовления 1,0 л 0,50 М уксусной кислоты требуется 50 мл 10 М уксусной кислоты.

    Рецепты общих растворов

    Для приготовления этих растворов медленно добавьте необходимые ингредиенты в мерную колбу объемом 1 л, наполовину заполненную дистиллированной или деионизированной водой. Дайте ингредиентам полностью раствориться, при необходимости осторожно покачивая колбу. Как только растворенное вещество полностью растворится и раствор достигнет комнатной температуры, разбавьте водой до метки.Вставьте и зафиксируйте пробку и несколько раз переверните колбу для перемешивания.

    Каждый реагент в следующей таблице связан с нашим онлайн-каталогом для получения дополнительной информации и удобной покупки. Для жидкостей мы ссылались на размер 500 мл реагента или химиката ACS, упакованного в безопасную бутылку с пластиковым покрытием. Для гидроксида натрия мы указали размер химического реагента класса чистоты 500 г. Могут быть доступны другие размеры и сорта. Пожалуйста, обратитесь к нашим каталогам для получения дополнительной информации.

    Решите ли вы создавать собственные решения или покупать их у нас в готовом виде, в Carolina вы найдете все, что вам нужно.

    Загрузить руководство по подготовке решения

    Магазин химикатов

    Часто задаваемые вопросы о приготовлении раствора

    Кэти Оуэнс
    Координатор по управлению продуктами

    Март 2017


    1. Как приготовить ___ мл ___ молярного раствора?

    Пример: Как приготовить 100 мл 3M водного раствора гидроксида натрия?

    Определите требуемую молярность: M = 3M раствор = 3 моля NaOH / 1 л раствора

    Определите молярную массу: MM NaOH = 40 г / моль

    Перевести требуемый объем в литры: V = 100 мл x 1 л / 1000 мл = 0.1 л

    Необходимые граммы = V x M x MM NaOH

    0,1 л раствора x 3 моля NaOH / 1 л раствора x 40 г NaOH / 1 моль NaOH = 12 г NaOH

    Приготовьте раствор:

    Ключевые термины

    Молярность : Концентрация раствора как количество молей растворенного вещества на литр раствора.

    Раствор : Гомогенная смесь, состоящая из двух или более веществ.

    Раствор : Вещество, растворенное в растворе.

    Растворитель : Вещество, растворяющее растворенное вещество с образованием раствора (обычно жидкости).

    Концентрация : Относительное количество растворенного вещества и растворителя в растворе.

    Молярная масса : Масса в граммах на один моль элемента или соединения.

    Объемный процент раствора : Концентрация раствора в миллилитрах растворенного вещества на 100 мл раствора.

    Гидрат : Неорганическая соль, которая содержит молекулы воды как часть своей структуры или кристаллизации.

    Смешиваемый : способность веществ образовывать гомогенную смесь при смешивании.

    1. Выберите колбу на 100 мл, предпочтительно мерную.
    2. Взвесьте 12 граммов гидроксида натрия.
    3. Наполните колбу примерно наполовину деионизированной или дистиллированной водой.
    4. Добавьте в колбу гидроксид натрия.
    5. Перемешайте до растворения, используя стержень для перемешивания и пластину для перемешивания, или осторожно перемешивая.
    6. Добавьте воды до отметки 100 мл на стеклянной посуде и снова перемешайте.

    2. Как разбавить раствор?

    Пример: Как мне приготовить 500-миллилитровый раствор 3M HCl кислоты из 12 M HCl кислоты?

    M 1 = Начальная молярность = 12 M
    V 1 = Начальный объем =?
    M 2 = Конечная молярность = 3 M
    V 2 = Конечный объем = 500 мл

    Рассчитайте объем исходного раствора, необходимый для разбавления:

    M 1 V 1 = M 2 V 2
    (12 M) (V 1 ) = (3 M) (500 мл)
    V 1 = 125 мл

    125 мл 12 M HCl кислоты требуется для получения 500 мл 3 M HCl кислоты.

    Приготовьте раствор:
    Используя мерную колбу на 500 мл и мерный цилиндр, добавьте 125 мл концентрированной кислоты к 250 мл воды. Обязательно медленно перемешивайте, а затем добавьте оставшуюся воду, чтобы достичь конечного объема 500 мл.

    Примечание : Если вам нужно приготовить очень разбавленный раствор из исходного раствора, вам может потребоваться выполнить более одного разбавления. Создав более управляемый и менее концентрированный раствор, окончательная молярность будет более точной.См. Нашу инфографику о проведении серийных разведений.


    3. Как перевести процентную концентрацию в молярность?

    Пример: Какова молярность 10% раствора гидроксида калия?

    В 100 мл 10% раствора КОН содержится 10 г гидроксида калия.
    ММ КОН = 56 г / моль

    10 г КОН x 1 моль КОН / 56 г КОН = 0,18 моль КОН

    0,18 моль КОН / 100 мл раствора x 1000 мл раствора / 1 л раствора = 1,8 моль КОН / л раствора = 1.8 месяцев солн

    Советы учителю

    1. Гидраты

      Учитывайте формулу веса гидратов при расчете концентрации растворов. Вес формулы гидратов будет выше за счет дополнительных молекул воды. Присоединенные молекулы воды вносят воду в окончательный раствор.

    2. Всегда добавляйте кислоту в воду, а не наоборот.

      Смешивание сильных кислот и воды вызывает экзотермическую реакцию.Если вы добавите воду к кислоте, вы сначала получите очень концентрированную кислоту. Реакция выделяет столько тепла, что раствор может выкипеть из емкости. Когда вы добавляете кислоту в воду, раствор постепенно становится более концентрированным. Если он все же разбрызгается, будет больше воды и меньше вероятность контакта с кислотой.

    3. Стандартные решения

      Хранение складских решений может сэкономить деньги и место! При необходимости основной раствор можно легко разбавить рабочим раствором нужной концентрации.Каролина предлагает большое разнообразие химических сортов, размеров и концентраций.

    4. P ersonal P rotective E quipment

      Всегда используйте надлежащие средства индивидуальной защиты (PPE) при работе с химическими веществами. См. Паспорт безопасности для получения инструкций по безопасному использованию, хранению и обращению. Обычные СИЗ для лаборатории включают:

      • Защитные очки или защитные очки
      • Халаты и фартуки
      • Перчатки (в зависимости от обрабатываемого материала)
      • Маска для лица
    5. Дистиллированный или деионизированный?

      Деионизированная вода
      была обработана для удаления неорганических заряженных частиц (ионов).В процессе не удаляются молекулы или биологические загрязнители.

      Дистиллированная вода
      не содержит почти всех неорганических минералов, химикатов и биологических загрязнителей.

      Для большинства лабораторных применений достаточно деионизированной воды.


    Дополнительные ресурсы

    .

    Previous PostNextNext Post

    Добавить комментарий

    Ваш адрес email не будет опубликован. Обязательные поля помечены *